>> Åwªï±z¡A³X«È¡G µn¤J½×¾Â «ö³o¸Ìµù¥U §Ñ°O±K½X ¦b½u·|­û ¤å³¹·j´M ½×¾Â­·®æ  ¨Ï¥Î»¡©ú   


>>> §ä¤H¿i¤ú©Î¥DÃD»Pª©­±µLÃöªº¹Ï¤å
¶ð¤º´Óª«¶é ¡÷ [ ´ú¸Õ°Ï ] [ªð¦^] ¡÷ ÂsÄý¡GSample cases¡@ ¼Ð°O½×¾Â©Ò¦³¤º®e¬°¤wŪ¨ú 

 ¥Ø«e½×¾ÂÁ`¦b½u 16 ¤H¡A¥»¥DÃD¦@¦³ 1 ¤HÂsÄý¡C¨ä¤¤µù¥U·|­û 0 ¤H¡A³X«È 1 ¤H¡C¡@ [Ãö³¬¸Ô²Ó¦W³æ]
µoªí¤@½g·s¥DÃD ¦^ÂФ峹 ¶}±Ò¤@­Ó·s§ë²¼ ¡»¦¹¤å³¹³Q¾\Ū 4771 ¦¸¡»¡@¡@ÂsÄý¤W¤@½g¥DÃD ¡@­«·s¾ã²z¥»¥DÃD  ¾ðª¬Åã¥Ü¤å³¹¡@ÂsÄý¤U¤@½g¥DÃD
 * ¤å³¹¥DÃD¡G Sample cases   ¥»¤å³¹¦³°ÝÃD¡A¶Ç°eµu°T®§³ø§iµ¹ª©¥D  ¥[¨ì§Úªº³Ì·R&Ãöª`¥»¤å³¹  Åã¥Ü¥i¦C¦Lªºª©¥»  §â¥»¤å³¹¥[¨ì§Úªº³Ì·R   
JuanFe 

 

¸ê®Æ: ¦¹·|­û¥Ø«e¤£¦b½u¤W Male ¨°Às ¥Õ¦Ï®y
«Â±æ: 0
¾y¤O: ¾y¤O: 78957
¸gÅç: ¸gÅç: 35986
¨Ó¦Û: ¦t©z¤¤¡@blank
µo¤å: 1118 ½g
ºëµØ: 0 ½g
¦b½u: 47¤Ñ19®É22¤À57¬í
µù¥U: 2013/06/17
Message ¬d¬Ý¡@·j´M¡@³q°T¿ý¡@½Æ»s¡@¤Þ¥Î¡@¦^ÂФ峹¦^ÂС@[²Ä 16 ¼Ó]
 15 Case 15 scenario (shortness of breath)

Doorway information about patient

The patient is a 60-year-old man who comes to the emergency department due to acute shortness of breath.

Vital signs

. Temperature: 36.7¡¦C(98F)
. Blood pressure : 110/70 mmHg
. Pulse : 90 /min
. Respirations : 26/min

Basic differential diagnosis

. Pulmonary embolism
. Congestive heart failure
. Chronic obstructive pulmonary disease exacerbation
. Pneumonia
. Spontaneous pneumothorax
. Asthma exacerbation
. Anxiety/panic attack

¡X¡X¡X¡X¡X

Case 15 sim.pt. instruction

If the doctor asks you about anything other than these , just say ¡§ no ,¡¨ or provide an answer that a normal patient might give.

You are a 60-year-old man with acute shortness of breath.

History of present illness

. Onset 2 days ago
. Mild shortness of breath with strenuous work that has slowly worsened over the past year. but is now suddenly worse
. Not of breath with walking 1 block
. Occasional dry cough
. No fever , chills , or unusual fatigue
. No syncope or palpations
. No swelling in the legs
. Difficulty at night
. Worsen when lying down and better when sitting up

Do not volunteer this information unless asked :
You sleep on 2 pillows because you get short of breath when lying flat , also , you woke up in the middle of the night weigh shortness of breath and could not go back to sleep for 20 minutes.

Past medical history

. Spinal fusion surgeries 2 weeks ago for spinal stenosis that required 1 week in the hospital postoperatively
. Hypertension diagnosed 20 years ago
. Medications ; hydrochlorothiazide 25mg daily
. Medication allergies : None
. Father died of a heart attack at age 55; mother is alive and has hypertension ; no siblings
. Occupation : Computer software analyst
. Married , live with wife
. Tobacco ; 1 pack of cigarettes a day for 40 years
. Alcohol : 1-2 glasses of wine a day for 35 years
. Recreational drugs : None

Physical examination

HEENT :
. PERRLA , EOMI

Neck :
. Supple without lymphadenopathy
. No JVD
. No thyromegaly

Lungs :
. Clear to auscultation and percussion

Heart : . Regular rate and rhythm
. No murmurs , gallops , or rubs

Extremities : . No edema , cyanosis , or clubbing
. Pedal pulse 2+ bilaterally
. No calf tenderness to palpation

¡X¡X¡X¡X¡X

Case 15 sim. pt. checklist

Following the encounter , check which of the following items were performed by the examinee

History of present illness/ review of systems

. Asked about the onset of symptoms
. Asked about the course over time
. Asked about associated symptoms :
- Chest pain
- Orthopnea and paroxysmal nocturnal dyspnea
- Wheezing
- Coughing
- Fever and chills
- Palpitations
- Syncope
- Leg pain and swelling
. Asked about recent travel or prolonged immobilization (recent surgery)

Past medical /family/social history

. Asked about similar episode in the past
. Asked about past medical issue (especially high blood pressure ,heart problems , asthma , and chronic obstructive pulmonary disease)
. Asked about surgeries and hospitalizations
. Asked about current medications
. Asked about medication allergies
. Asked about occupation
. Asked about tobacco , alcohol , and drug use
. Asked about family history of blood clots and heart problems

Examination

. Washed heads before examination
. Examined without gown , not through gown
. Examined neck for jugular venous pressure
. Examined heart : inspection , palpation , auscultation
. Examined lungs : inspection , palpation , auscultation , percussion
. Examined the extremities for pulses and edema
. Checked calf muscle tenderness

Counseling

. Explained physical findings and possible diagnosis
. Explained further workup
. Discussed quitting smoking

Communication skills and professional conduct

. Knocked before entering the room
. Introduced self and greeted you warmly
. Used your name to address you
. Paid attention to what you said and maintained good eye contact
. Asked opened questions
. Asked non-leading questions . Asked one question at a time
. Listened to what you said without interrupting me
. Used plain English rather than technical jargon
. Used appropriate transition sentences
. Used appropriate draping techniques
. Summarized the history and explained physical findings
. Expressed empathy and gave appropriate reassurances
. Asked whether you have any concerns/questions

Differential diagnosis

. Pulmonary embolism
. Pneumonia
. Congestive heart failure
. Chronic obstructive pulmonary disease

Diagnostic study/studies

. CBC with differential count
. Chest x-ray
. ECG
. Ventilation / perfusion (V/Q) scan or  chest CT scan
. Echocardiogram

¡X¡X¡X¡X¡X

Case 15 clinical summary

Clinical Skills Evaluation
Case  Patient Note

The following represents a typical note for this patient encounter . the detail may vary depending on the information given by the simulated patient.

History : Describe the history you just obtained form this patient . Include only information (Pertinent positives and negatives)  relevant to this patient¡¦s problem(s).

. 60-yo man with 2 days of worsening shortness of breath.
. 1 years of gradually worsening shortness of breaths that acutely worsened 2 days ago.
. New 2 -pillow orthopnea , PND.
. Spinal fusion surgery 2 weeks ago.
. Occasional dry cough.

ROS: No chest pain , wheezing , fatigue , palpations , leg swelling , syncope , fever , or chills
PMHx : HTN
PSHx : Spinal fusion surgery
Meds : Hydrochlorothiazide 25 mg daily
Allergies : None
FHx: Father died of MI at age 55 , mother has HTN
SHx : 1 PPD smoker for 40 years , 1 or 2 glasses of wine /day for 35 years

Physical examinations : Describe any positive and  negative findings relevant  to this patient¡¦s problem(s) . be careful to include only those parts of the examination performed in this encounter.

. Vital signs: Temperature , 36.7¡¦C(98.1F) ; blood pressure , 110/70 mmHg; pulse , 90/min; and respiration , 26/min
. HEENT : PERRLA , EMOI , no jaundice
. Neck ; Supple without lymphadenopathy , no JVD , no thyromegaly
. Lungs ; clear to auscultation
. Heart : RRR without murmurs, gallops , or rubs
. Extremities ; No edema , pulse 2+ bilaterally , no cyanosis or clubbing , no lower extremities tenderness on palpation

Data interpretation : Based on what you have learned from the history and physical examination, list up to 3 diagnosis that might explain this patient¡¦s complaint(s) . List your diagnoses from most to least likely . For some cases , fewer than 3 diagnosis will be appropriate . the , enter the positive and negative findings form the history and the physical examination (if present) that support each diagnosis . Finally , list initial diagnostic (if any) you would order for each listed diagnosis (eg,restricted physical examination maneuvers, laboratory tests , imaging ECG , etc).

Diagnosis #1 :Pulmonary embolism

History finding(s)
. Acute worsening of dyspnea
. Tecent spinal fusion surgery

Physical examination finding(s)
. None

Diagnosis #2 : Congestive heart failure

History finding(s)
. 1 year of gradually worsening dyspnea
. Dyspnea worse with exertion
. Orthopnea and PND

Physical examination finding(s)
. None

Diagnosis #3 : COPD

History finding(s)
. History of smoking
. 1 year of gradually worsening dyspnea
. Cough

Physical examination finding(s)
. None

Diagnostic studies
. Chest x-ray
. CT anagram of chest
. CBC with differential
. Basic metabolic panel
. ECG , echocardiogram







µoªí¤å³¹®É¶¡2018/08/15 03:32pm¡@IP: ¤w³]©w«O±K[¥»¤å¦@ 7910 ¦ì¤¸²Õ]¡@ 
JuanFe 

 

¸ê®Æ: ¦¹·|­û¥Ø«e¤£¦b½u¤W Male ¨°Às ¥Õ¦Ï®y
«Â±æ: 0
¾y¤O: ¾y¤O: 78957
¸gÅç: ¸gÅç: 35986
¨Ó¦Û: ¦t©z¤¤¡@blank
µo¤å: 1118 ½g
ºëµØ: 0 ½g
¦b½u: 47¤Ñ19®É22¤À57¬í
µù¥U: 2013/06/17
Message ¬d¬Ý¡@·j´M¡@³q°T¿ý¡@½Æ»s¡@¤Þ¥Î¡@¦^ÂФ峹¦^ÂС@[²Ä 17 ¼Ó]
 16 Case 16 scenario (increase urination)

Doorway information about patient

The patient is a 40-year-old woman who comes to the office due to increased urination

Vital signs

. Temperature : 36.7¡¦C (98F)
. Blood pressure : 110/70 mmHg
. Pulse : 86/min
. Respirations : 16/min

Basic differential diagnosis

Increased urine volume
. Diabetes mellitus
. Diabetes insipidus (central, nephrogenic)
. Psychogenic : polydipsia
. Diuretic use
. hypercalcemia

increased urinary frequency
. Urinary tract infection
. Overactive bladder
. Excess caffeine intake
. Vaginitis , urethritis

¡X¡X¡X¡X¡X

Case 16 sim.pt. instruction

If the doctor asks you about anything other than these , just say ¡§ no ,¡¨ or provide an answer that a normal patient might give.

You are a 40-year-old women complaining of increased urinary frequency

History of present illness

. Onset 2  months ago
. Urination 8-10 times during the day and 2-3 times a night
. increased urinary volume
. Increased feelings of thirst for last month
. Associated symptoms: - Fatigue
- 4.5-kg (10-lb) weight loss (despite increased appetite)
- No dysuria or urgency
- No fever or chills

Past medical/family/soical history

. Bipolar disorder diagnosis 20 years ago
. Minor head injury after falling off bicycle 3 months ago ; seen in emergency department and discharged without intervention
. No surgeries or hospitalizations
. Medications : lithium 60 mg twice daily
. Medication allergies : None
. Married , live with husband
. 2 pregnancies with normal vaginal delivery ; both children are healthy
. Both parents have type 2 diabetes mellitus l no siblings
. Tobacco : No
. Alcohol : No
. Recreational drugs : No

Physical examination

HEENT :
. PERRLA , EOMI
. Visual fields intact

Heart :
. Regular rate and rhythm
. No murmurs , gallops , or rubs

Abdomen :
. Soft and non-tender with normal bowel sounds
. No suprapubic or CVA tenderness

Neurologic :
. Muscle strength 5/5 throughout
. Sensation in tact in all 4 extremities
. reflexes 2+ in all 4 extremities

¡X¡X¡X¡X¡X

Case 16 sim. pt. checklist

Following the encounter , check which of the following items were performed by the examinee

History of present illness/ review of systems

. Asked about the onset of problem
. Asked about the frequency of urination
. Asked about nocturia
. Asked about nocturne
. Asked about urine volume
. Asked about burning on urination
. Asked about urgency and hesitancy of urination
. Asked boy increased thirst and fluid intake
. Asked about appetite  and changes in weight
. Asked about the trauma to the head

Past medical /family/social history

. Asked about similar problems in the past
. Asked about past medical issues , hospitalizations ,and surgeries
. Asked about psychiatric problems (history of bipolar disorder , schizophrenia)
. Asked about current medications
. Asked about family health(especially diabetes)
. Asked about tobacco , alcohol , and drug use
. Asked about occupation

Examination

. Washed heads before examination
. Examined without gown , not through gown
. Examined mucous membranes
. Examined heart and lungs
. Tested muscle power in both upper and lower limbs
. Tested sensation in the lower extremities
. Tested reflexes in the lower extremities
. Tested visual fields and examined funds
. Tested for suprapubic and costovertebral angle tenderness

Counseling

. Explained physical findings and possible diagnosis
. Explained further workup

Communication skills and professional conduct

. Knocked before entering the room
. Introduced self and greeted you warmly
. Used your name to address you
. Paid attention to what you said and maintained good eye contact
. Asked opened questions
. Asked non-leading questions . Asked one question at a time
. Listened to what you said without interrupting me
. Used plain English rather than technical jargon
. Used appropriate transition sentences
. Used appropriate draping techniques
. Summarized the history and explained physical findings
. Expressed empathy and gave appropriate reassurances
. Asked whether you have any concerns/questions

Differential diagnosis

. Diabetes mellitus
. Central diabetes insipidus
. Nephrogenic diabetes insidious (lithium side effect)
. Psychogenic polydipsia
. Hypercalcemia

Diagnostic study/studies

. Fasting blood sugar
. Urinalysis
. Serum electrolytes (Na,K, Cl , CO2 , BUN , Cr , and calcium)
. Urine and serum osmolality

¡X¡X¡X¡X¡X

Case 16 clinical summary

Clinical Skills Evaluation
Case  Patient Note

The following represents a typical note for this patient encounter . the detail may vary depending on the information given by the simulated patient.

History : Describe the history you just obtained form this patient . Include only information (Pertinent positives and negatives)  relevant to this patient¡¦s problem(s).

. 40-yo woman with 2 months of polyuria , polydipsia, nocturia , and polyphagia
. 2-3 month of 4.5-kg(10-lb) weight loss with fatigue
. No dysuria or urinary urgency

ROS : No fever or chills
PMHx : bipolar disorder diagnosed 20 years ago ; minor head trauma 3 months ago , seen in emergency department and discharged without intervention
PSHx : None
Meds : Lithium 600 mg 2 times daily
Allergies : None
FHx : Father and mother have diabetes
SHx : No history of tobacco or alcohol use

Physical examinations : Describe any positive and  negative findings relevant  to this patient¡¦s problem(s) . be careful to include only those parts of the examination performed in this encounter.

Vital signs: Temperature , 36.7¡¦C(98.1F) ; blood pressure , 110/70mmHg; pulse , 86/min; and respirations , 16/min
. HEENT : PERRLA , EOMI, intact visual fields
. Abdomen : Non-tender without suprapubic tenderness, np CVA tenderness
. Neurologic L muscle strength 5/5 throughout sensation grossly intact bilateral lower extremities , DTR 2 + in bilateral lower extremities

Data interpretation : Based on what you have learned from the history and physical examination, list up to 3 diagnosis that might explain this patient¡¦s complaint(s) . List your diagnoses from most to least likely . For some cases , fewer than 3 diagnosis will be appropriate . the , enter the positive and negative findings form the history and the physical examination (if present) that support each diagnosis . Finally , list initial diagnostic (if any) you would order for each listed diagnosis (eg,restricted physical examination maneuvers, laboratory tests , imaging ECG , etc).

Diagnosis #1 : Diabetes mellitus

History finding(s)
. Polyuria
. Polydipsia and polyphagia
. Family history of diabetes in father and mother
. Weight loss

Physical examination finding(s)
. None

Diagnosis #2 : Diabetes insipidus

History finding(s)
. History of bipolar disorder
. Lithium use
. Polyuria

Physical examination finding(s)
. None

Diagnosis #3 : Psychogenic polydipsia

History finding(s)
. history of bipolar disorder
. Polyuria
. Polydipsia

Physical examination finding(s)
. None

Diagnostic studies
. Fasting blood glucose
. Hemoglobin A1c
. Urinalysis
. Serum electrolytes , lithium level
. Urine and serum osmolality







µoªí¤å³¹®É¶¡2018/08/15 03:33pm¡@IP: ¤w³]©w«O±K[¥»¤å¦@ 7569 ¦ì¤¸²Õ]¡@ 
JuanFe 

 

¸ê®Æ: ¦¹·|­û¥Ø«e¤£¦b½u¤W Male ¨°Às ¥Õ¦Ï®y
«Â±æ: 0
¾y¤O: ¾y¤O: 78957
¸gÅç: ¸gÅç: 35986
¨Ó¦Û: ¦t©z¤¤¡@blank
µo¤å: 1118 ½g
ºëµØ: 0 ½g
¦b½u: 47¤Ñ19®É22¤À57¬í
µù¥U: 2013/06/17
Message ¬d¬Ý¡@·j´M¡@³q°T¿ý¡@½Æ»s¡@¤Þ¥Î¡@¦^ÂФ峹¦^ÂС@[²Ä 18 ¼Ó]
 17 Case 17 scenario (jaundice)

Doorway information about patient

The patient is a 35-year-old woman who comes to the office due to jaundice

Vital signs

. Temperature : 38.5¡¦C (101.3F)
. Blood pressure : 120/75 mmHg
. pulse : 98/min
. Respirations : 22/min

Basic differential diagnosis

. Infectious hepatitis
. Hemolytic jaundice . Alcoholic hepatitis
. Drug-induced hepatitis
. Primary biliary cirrhosis
. Wilson disease
. Hemochromatosis
. Malignancy

¡X¡X¡X¡X¡X

Case 17 sim.pt. instruction

If the doctor asks you about anything other than these , just say ¡§ no ,¡¨ or provide an answer that a normal patient might give.

You are a 35-year-old woman who comes to the clinic due to jaundice

History of present illness

. Onset of jaundice 2 days ago
. Fever beginning 5 days ago
. Dark urine and pale stools ,then yellowing of the eyes
. Associated symptoms :
- Mild abdomen discomfort
- 2-3 episodes non bilious , non bloody vomiting
- No sore throat , headache , neck stiffness , itching , diarrhea , or constipation
. Recent return form a trip to india (ate local food and drank bottled water)
. No vaccination for hepatitis in the past

Past medical history

. Motor vehicle accident 2 years ago requiring hospitalization and blood transfusion
. No surgires
. No medications
. No medication allergies
. Immediate family members 9parents and 3 siblings ) are all healthy
. Occupation: homemaker
. Married , live with husband
. 2 pregnancies with uncomplicated delivers
. Tobacco : no
. Alcohol  : 2-3 beers a day for 15 years
. recreational drugs : No

Physical examination

HEENT
. PERRLA, EOMI
. Scleral icterus present
. Oropharynx clear

Neck :
. No lymphadenopathy

Abdomen :
. Non-tender , non-distended
. No hepatosplenomegaly
. Normative bowel sounds

¡X¡X¡X¡X¡X

Case 17 sim. pt. checklist

Following the encounter , check which of the following items were performed by the examinee

History of present illness/ review of systems

. Asked about the onset and duration of jaundice
. Asked about the color of stool and urine
. Asked about itching /pruritis
. Asked about abdominal pain
. Asked about fever and chills
. Asked about appetite and changes in weight
. Asked about sore throat
. Asked about any bleeding tendencies
. Asked about enlarged glands
. Asked about travel history

Past medical /family/social history

. Asked about similar episodes before
. Asked about past medical issue (especially hepatitis ,liver disease , blood transfusion , high blood pressure , diabetes ) , hospitalizations , and surgeries
. Asked about current medications
. Asked about family health
. Asked bout tobacco , alcohol , and drug use
. Asked bout sexual and reproductive history . Asked bout occupation

Examination

. Washed heads before examination
. Examined without gown , not through gown
. Auscultation abdomen
. Palpated abdomen (superficial abandon deep) , including liver and spleen
. Checked rebound tenderness
. Examined for enlarged nodes

Counseling

. Explained physical findings and possible diagnosis
. Explained further workup
. Explained the importance of lifestyle modifications (especially reducing alcohol intake)

Communication skills and professional conduct

. Knocked before entering the room
. Introduced self and greeted you warmly
. Used your name to address you
. Paid attention to what you said and maintained good eye contact
. Asked opened questions
. Asked non-leading questions . Asked one question at a time
. Listened to what you said without interrupting me
. Used plain English rather than technical jargon
. Used appropriate transition sentences
. Used appropriate draping techniques
. Summarized the history and explained physical findings
. Expressed empathy and gave appropriate reassurances
. Asked whether you have any concerns/questions

Differential diagnosis

. infectious hepatitis
. Alcohol hepatitis
. Drug-induced hepatitis
. Primary biliary cirrhosis
. Malignancy

Diagnostic study/studies

. CBC with differential
. Liver function tests (AST,ALT, alkaline phosphatase , bilirubin)
. Coagulation studies (PT,PTT)
. Viral hepatitis serologies ( HBs antigen , HBc antibody , hepatitis A antibody , hepatitis C antibody)
. Urine for bile salts
. Anti-mitochondrial antibodies
. Liver ultrasound

¡X¡X¡X¡X¡X

Case 17 clinical summary

Clinical Skills Evaluation
Case 17 Patient Note

The following represents a typical note for this patient encounter . the detail may vary depending on the information given by the simulated patient.

History : Describe the history you just obtained form this patient . Include only information (Pertinent positives and negatives)  relevant to this patient¡¦s problem(s).

. 35-yo woman with 2 days of jaundice associated with fever
. Returned 3 weeks ago form trip to india
. Dark urine endplate stools
. Mild diffuse abdominal discomfort , nausea ,vomiting.

ROS : No sore throat , headache , stiff neck , dysuria , weight loss , itching , diarrhea , or constipation
PMHx: Car accident 2 years ago  requiring hospitalization and blood transfusion
PSHx: None
Meds: None
Allergies : None
FHx: Father , mother and siblings are healthy
SHx: No history of tobacco use , 2-3 beers / day for the past 15 years

Physical examinations : Describe any positive and  negative findings relevant  to this patient¡¦s problem(s) . be careful to include only those parts of the examination performed in this encounter.

. Vital sign: temperature , 38.5¡¦C (101.3F) ; blood pressure , 120/75 mmHg; pulse , 98/min; respirations , 22/min
. HEENT : PERRLA , EOMI , icterus present , oropharynx clear
. Neck : No lymphadenopathy
. Abdomen : Non-tender and non-distended , no hepatosplenomegaly , normative bowel sounds

Data interpretation : Based on what you have learned from the history and physical examination, list up to 3 diagnosis that might explain this patient¡¦s complaint(s) . List your diagnoses from most to least likely . For some cases , fewer than 3 diagnosis will be appropriate . the , enter the positive and negative findings form the history and the physical examination (if present) that support each diagnosis . Finally , list initial diagnostic (if any) you would order for each listed diagnosis (eg,restricted physical examination maneuvers, laboratory tests , imaging ECG , etc).

Diagnosis #1 : infectious hepatitis

History finding(s)
. Travel to India
. Jaundice
. Nausea , vomiting , abdomen discomfort
. Dark urine , pale stools

Physical examination finding(s)
. Fever . Jaundice

Diagnosis #2 : Alcoholic hepatitis

History finding(s)
. Alcohol use
. Nausea , vomiting , abdominal discomfort

Physical examination finding(s)
. Fever
. Jaundice

Diagnostic studies
. Hepatitis serologies : A,B, and C
. CBC with differential
. Liver function tests
. Urinalysis








µoªí¤å³¹®É¶¡2018/08/15 03:34pm¡@IP: ¤w³]©w«O±K[¥»¤å¦@ 7196 ¦ì¤¸²Õ]¡@ 
JuanFe 

 

¸ê®Æ: ¦¹·|­û¥Ø«e¤£¦b½u¤W Male ¨°Às ¥Õ¦Ï®y
«Â±æ: 0
¾y¤O: ¾y¤O: 78957
¸gÅç: ¸gÅç: 35986
¨Ó¦Û: ¦t©z¤¤¡@blank
µo¤å: 1118 ½g
ºëµØ: 0 ½g
¦b½u: 47¤Ñ19®É22¤À57¬í
µù¥U: 2013/06/17
Message ¬d¬Ý¡@·j´M¡@³q°T¿ý¡@½Æ»s¡@¤Þ¥Î¡@¦^ÂФ峹¦^ÂС@[²Ä 19 ¼Ó]
 18 Case 18 scenario (chest pain)

Doorway information about patient

The Patient is a 35-yea-old woman who comes to the emergency department due to chest pain

Vital signs

. Temperature ; 36.8¡¦C (98.3F)
. Blood pressure : 120/75 mmHg
. Pulse : 98/min
. Respirations : 12 /min

Basic differential diagnosis
. Pneumonia . Gastroesphageal reflux disease
. Pain disorder/ hyperventilation syndrome
. hyperthyroidism
. Angina
. Costochondritis

¡X¡X¡X¡X¡X

Case 18 sim.pt. instruction

If the doctor asks you about anything other than these , just say ¡§ no ,¡¨ or provide an answer that a normal patient might give.

You are a 35-year-old woman with chest pain

History of present illness

. Onset of this episode 1 days ago
. Squeezing sensation over the entire chest
. 4-5/10 severity
. No aggravating or relieving factors ( no relief form antacids)
. Intermittent chest discomfort for the last year; episodes usually last 20 minutes ; initially about once a month but now every week; usually triggered buy going out in public
. Associated symptoms:
- Fasting breathing
- Sweating - Headache
- Palpations
. Hospitalized over might 6 months ago for the same problem; all tests came back normal

Review of systems

. No nausea , vomiting , diarrhea , or abdominal pain
. No dysuria
. Headache
. Palpitations

Past medical history

. Hospitalized following a motor vehicle accident 32 years ago (required blood transfusion)
. No surgeries
. Medications : None
. Medication allergies : None
. After os healthy ; mother has ¡§hypochondriasis¡¨ ; sister has hyperthyroidism
. Occupation : Secretary at a law firm
. Single, live a lone
. Tobacco : 1 pack a day for 15 years
. Alcohol : None
. Recreational drugs ; None

Physical examination

Neck :
. No lymphadenopathy
. No thyromegaly

Lungs :
. Clear to auscultation bilaterally

Heart :
. Regular rate and rhythm
. No murmurs, gallops , or rubs

Ask this question : ¡§Do you think that this is a heart attack ? I feel like I am going to die ¡§

¡X¡X¡X¡X¡X

Case 18 sim. pt. checklist

Following the encounter , check which of the following items were performed by the examinee

History of present illness/ review of systems

. Asked about the location of pain
. Asked about the intensity of pain
. Asked about the quality of pain
. Asked about the duration and frequency of pain
. Asked about the course of the painter time
. Asked about any radiation of pain
. Asked about any aggravating or relieving factors
. Asked about any precipitating factors
. Asked about associated symptoms
- Nausea and vomiting
- Sweating
- hyperventilation or trouble breathing
- Cough
- Palpitations or rapid heart beat
- Fear of dying or sense of tremor
- Syncope or dizziness
- Headache
- changes in appetite or weight

Past medical /family/social history

. Asked about similar episodes in the past
. Asked about past medical issues (especially high blood pressure , heart problems , diabetes , thyroid problems)
. Asked about previous hospitalizations and surgeries
. Asked about current medications
. Asked about medication allergies
. Asked about family health
. Asked about tobacco , alcohol , and drug use
. Asked about sexual and reproductive history
. Asked bout occupation and stress in life

Examination

. Washed heads before examination
. Examined without gown , not through gown
. Examined neck and thyroid
. Listened to heart and lungs
. Check reflexes

Counseling

. Explained physical findings and possible diagnosis
. Discussed result of chest x-ray
. Explained further workup
. Discussed quitting smoking

Communication skills and professional conduct

. Knocked before entering the room
. Introduced self and greeted you warmly
. Used your name to address you
. Paid attention to what you said and maintained good eye contact
. Asked opened questions
. Asked non-leading questions . Asked one question at a time
. Listened to what you said without interrupting me
. Used plain English rather than technical jargon
. Used appropriate transition sentences
. Used appropriate draping techniques
. Summarized the history and explained physical findings
. Expressed empathy and gave appropriate reassurances
. Asked whether you have any concerns/questions

Differential diagnosis

. Panic disorder
. Hyperthyroidism
. Hyperventilation syndrome
, Angina

Diagnostic study/studies

. CBC with differential
. Electrolytes and blood glucose
. ECG
. TSH

¡X¡X¡X¡X¡X

Case 18 clinical summary

Clinical Skills Evaluation
Case 18 Patient Note

The following represents a typical note for this patient encounter . the detail may vary depending on the information given by the simulated patient.

History : Describe the history you just obtained form this patient . Include only information (Pertinent positives and negatives)  relevant to this patient¡¦s problem(s).

. 35-yo woman with 1 year of worsening episodes of chest pain
. Diffuse chest tightness associated with headache , palpations , diaphoresis , hyperventilation ,a dn sense of impending doom
. Previous hospitalization with normal testing
. Episodes occur more often when out in public and are not relieved with antacids.

ROS : No nausea , vomiting , dysuria , constipation , flushing , sleep disturbance , or abdominal pain
PMHx: None
PSHx: None
Meds: None
Allergies : None
FHx : Fater os healthy , mother has hypochondriasis , and sister has hyperthyroidism
SHx : 1 PPD smoker for 15 years , no use of alcohol or illicit drugs

Physical examinations : Describe any positive and  negative findings relevant  to this patient¡¦s problem(s) . be careful to include only those parts of the examination performed in this encounter.

. Vital signs: Temperature , 36.8¡¦C (98.3F); blood pressure , 120/75mmHg; pulse , 98/min; and respirations, 12/min
. Neck : No lymphadenopathy por thyromegaly
. Lungs ; Clear to auscultation bilaterally
. heart : RRR without murmurs, gallops, or rubs

Data interpretation : Based on what you have learned from the history and physical examination, list up to 3 diagnosis that might explain this patient¡¦s complaint(s) . List your diagnoses from most to least likely . For some cases , fewer than 3 diagnosis will be appropriate . the , enter the positive and negative findings form the history and the physical examination (if present) that support each diagnosis . Finally , list initial diagnostic (if any) you would order for each listed diagnosis (eg,restricted physical examination maneuvers, laboratory tests , imaging ECG , etc).

Diagnosis #1 : Panic disorder

History finding(s)
. Episodes of chest pain
. Worse when in public
. Normal investigation in the past

Physical examination finding(s)
. Normal vital signs
. Normla cardiac examination

Diagnosis #2 : Hyperthyroidism

History finding(s)
. Family history
. Palpitations

Physical examination finding(s)
.None

Diagnosis #3 : Cardiac arrthemia

History finding(s)
. Episodic palpations
. Chest pain and diaphoresis

Physical examination finding(s)
. None

Diagnostic studies
. ECG
. TSH
. Chest x-ray shows only mild hyperinflation







µoªí¤å³¹®É¶¡2018/08/15 03:35pm¡@IP: ¤w³]©w«O±K[¥»¤å¦@ 7508 ¦ì¤¸²Õ]¡@ 
JuanFe 

 

¸ê®Æ: ¦¹·|­û¥Ø«e¤£¦b½u¤W Male ¨°Às ¥Õ¦Ï®y
«Â±æ: 0
¾y¤O: ¾y¤O: 78957
¸gÅç: ¸gÅç: 35986
¨Ó¦Û: ¦t©z¤¤¡@blank
µo¤å: 1118 ½g
ºëµØ: 0 ½g
¦b½u: 47¤Ñ19®É22¤À57¬í
µù¥U: 2013/06/17
Message ¬d¬Ý¡@·j´M¡@³q°T¿ý¡@½Æ»s¡@¤Þ¥Î¡@¦^ÂФ峹¦^ÂС@[²Ä 20 ¼Ó]
 19 Case 19 scenario (abdominal pain RLQ)

Doorway information about patient

The patient is a 45-year-old man who comes to the emergency department due to right lower abdominal pain

Vital signs

. Temperature : 37.1¡¦C (89.7F)
. Blood pressure : 130/80 mmHg
. Pulse : 100/min
. Respirations : 20/min

Basic differential diagnosis

. Appendicitis
. Mickel diverticulitis
. Perforation viscus
. Intestinal obstruction
. Yersinia enterocolitica
. Pancreatitis
. Urolithiasis
. Acute cholecystitis
. Herpis zoster (shingle)

¡X¡X¡X¡X¡X

Case 19 sim.pt. instruction

If the doctor asks you about anything other than these , just say ¡§ no ,¡¨ or provide an answer that a normal patient might give.

You are a 45-year-old man who comes to the emergency department with abdominal pain

History of present illness

. The pain stated suddenly 2 hours ago
. Pain started behind the belly button , than radiated to the right lower abdomen
. Pain is sharp. 8-9/10 severity , and has increased over time
. The symptoms started after a large meal and are worse with movement ; nothing relives the pain
. Associated symptoms
-  Nausea
- 2 episodes of non bilious , non bloody vomiting
. Last bowel movement was 20 hours ago , and you are passing gas normally

Review of systems

. No fever or chill
. No dysuria
. No diarrhea or constipation
. No back pain

Past medical/family/social history

. Peptic ulcer disease ; treated 10 years ago with omeprazole
. No current medications
. No surgeries
. Immediate family members are all healthy
. Occupation : Bus driver
. Single ,live alone
. Tobacco : 1 pack a day for 20 years
. Alcohol : 3 beer a day for past 15 years
. Recreational drugs : No

Physical examination

Abdomen :
. Right lower quadrant tenderness to superficial and deep palpation
. Rebound tenderness noted
. Normative bowel sounds throughout
. No hepatosplenomegaly
. No CVA tenderness
. Posts and obturator signs : Negative

¡X¡X¡X¡X¡X

Case 19 sim. pt. checklist

Following the encounter , check which of the following items were performed by the examinee

History of present illness/ review of systems

. Asked about the location and radiation of pain
. Asked about the intensity of pain
. Asked about the quality of pain
. Asked about the origin and duration of pain
. Asked about the course of pain over time
. Asked about any aggravating or relieving factors
. Asked about any vomiting
. Asked about fever
. Asked bout urinary problems
. Asked about bowel problems , constipation , and last bowel movement
. Asked about appetite and change in weight

Past medical /family/social history

. Asked about similar episodes  in the past
. Asked about past medical issues, hospitalizations , and surgeries
. Asked about current medications
. Asked about ,medical allergies
. Asked about family health
. Asked about tobacco , alcohol ,and recreational drug use
. Asked about sexual history
. Asked occupation

Examination

. Washed heads before examination
. Examined without gown , not through gown
. Auscultated abdomen(before palpation)
. Palpated abdomen(superficial and deep)
. Checked rebound tenderness
. Check for costovertebral angle tenderness
. Percussed for liver span
. Performed psoas sign and obturator sign

Counseling

. Explained physical findings and possible diagnosis
. Explained further workup
. Discussed lifestyle modifications , including quitting smoking and reducing alcohol

Communication skills and professional conduct

. Knocked before entering the room
. Introduced self and greeted you warmly
. Used your name to address you
. Paid attention to what you said and maintained good eye contact
. Asked opened questions
. Asked non-leading questions . Asked one question at a time
. Listened to what you said without interrupting me
. Used plain English rather than technical jargon
. Used appropriate transition sentences
. Used appropriate draping techniques
. Summarized the history and explained physical findings
. Expressed empathy and gave appropriate reassurances
. Asked whether you have any concerns/questions

Differential diagnosis

. Appendicitis
. Meckel diverticulitis
. Perforated peptic ulcer
. Intestinal obstruction
. Pancreatitis
. Urolithiasis

Diagnostic study/studies

. CBC with differential
. Serum chemistries(glucose, electrolytes , liver enzymes , creatinine)
. Serum lipas
. Abdomen x-ray
. Abdomen ultrasound
. Lipase, amylase
. Upper GI endoscopy

¡X¡X¡X¡X¡X

Case 19 clinical summary

Clinical Skills Evaluation
Case 19 Patient Note

The following represents a typical note for this patient encounter . the detail may vary depending on the information given by the simulated patient.

History : Describe the history you just obtained form this patient . Include only information (Pertinent positives and negatives)  relevant to this patient¡¦s problem(s).

. 45-yo man with 2 hors of worsening poplin form mid-epigastric region to RLQ
. Pain worse with movement but not relieved buy anything
. Nausea and vomiting (non-bloody , non-bilious)
. Last bowel movement 20 hours ago with passage flatus.

ROS : No fever , chills , diarrhea , constipation , or back pain
PMHx : Peptic ulcer disease
PSHx: None
Meds : None
Allergies : None
FHx : Father ,mother , and siblings are healthy
SHx : 1 PPD smoker for past 20 years , 3 beers .day for past 15 years

Physical examinations : Describe any positive and  negative findings relevant  to this patient¡¦s problem(s) . be careful to include only those parts of the examination performed in this encounter.

. Vital signs : Temperature, 37.1¡¦C(98.8F) ; blood pressure , 130/80mmHg ; pulse ,100/min ; and respirations , 20/min
. Abdomen ; RLQ tenderness to superficial and deep palpation , rebound tenderness present , normative bowel sounds throughout , no hepatosplenomegaly, no CVA tenderness ,negative psoas and obstructor signs Data interpretation : Based on what you have learned from the history and physical examination, list up to 3 diagnosis that might explain this patient¡¦s complaint(s) . List your diagnoses from most to least likely . For some cases , fewer than 3 diagnosis will be appropriate . the , enter the positive and negative findings form the history and the physical examination (if present) that support each diagnosis . Finally , list initial diagnostic (if any) you would order for each listed diagnosis (eg,restricted physical examination maneuvers, laboratory tests , imaging ECG , etc).

Diagnosis #1 : Appendicitis

History finding(s)
. Sudden onset RLQ pain
. Nausea and vomiting

Physical examination finding(s)
. RLQ tenderness
. Rebound tenderness in abdomen

Diagnosis #2 : Perforated peptic ulcer

History finding(s)
. History of peptic ulcer
. Abdomen pain
. Alcohol use

Physical examination finding(s)
. Rebound tenderness in abdomen

Diagnosis #3 : Intestinal obstruction

History finding(s)
. Abdomen pain
. Last bowel movement 20 hors ago

Physical examination finding(s)
. Rebound tenderness in abdomen

Diagnostic studies
. CBC with differential
. Abdominal x-ray
. CT of the abdomen







µoªí¤å³¹®É¶¡2018/08/15 03:36pm¡@IP: ¤w³]©w«O±K[¥»¤å¦@ 7481 ¦ì¤¸²Õ]¡@ 
JuanFe 

 

¸ê®Æ: ¦¹·|­û¥Ø«e¤£¦b½u¤W Male ¨°Às ¥Õ¦Ï®y
«Â±æ: 0
¾y¤O: ¾y¤O: 78957
¸gÅç: ¸gÅç: 35986
¨Ó¦Û: ¦t©z¤¤¡@blank
µo¤å: 1118 ½g
ºëµØ: 0 ½g
¦b½u: 47¤Ñ19®É22¤À57¬í
µù¥U: 2013/06/17
Message ¬d¬Ý¡@·j´M¡@³q°T¿ý¡@½Æ»s¡@¤Þ¥Î¡@¦^ÂФ峹¦^ÂС@[²Ä 21 ¼Ó]
 20 Case 20 scenario (leg pain bilateral)

Doorway information about patient

The patient is a 50-year-old man who comes to the office due to bilateral leg pain

Vital signs

. Temperature : 36.7¡¨C(98F)
. Blood pressure : 140/80 mmHg
. Pulse : 78/min
. Respirations : 20 /min

Basic differential diagnosis

Bilateral pain
. Atherosclerotic vascular disease
. Lumber spinal stenosis
. Diabetic polyneuropathy
. Radiculopathy due to spinal disease
. Medications , such as statin
. Trauma
. Thromboangiitis obliterans

Unilateral pain
. Cellulitis / myofasciitis
. Deep vein thrombosis
. Rupture of baker cyst
. Osteomyelitis
. Radiculopathy /sciatica
. Pathological fracture of the bone

¡X¡X¡X¡X¡X

Case 20 sim.pt. instruction

If the doctor asks you about anything other than these , just say ¡§ no ,¡¨ or provide an answer that a normal patient might give.

You are a 50 year-old-man with bilateral leg pain

History of present illness

. Onset 2 months ago
. Pain stated gradually and has become progressively worse
. Throbbing pain with 5-6 /10 severity
. Located in the calf muscles , no radiation
. Worse with walking , running , and prolonged standing
. Better while sitting and at rest
. No history of trauma

Review of systems

. No fever
. No back pain
. No weakness , numbness , or tingling in the leg
. No sexual or bladder symptoms

Past medical / Family / social history

. Diabetes for the past 3 years , controlled by diet
. High cholesterol
. No surgires
. Medications : Simvastain 40 mg  daily at bedtime
. Allergies : No
. Father died at age 65 of a stroke ; mother and 2 sibling are healthy
. Occupation: Postal worker
. Married , live with wife
. Tobacco : 2 pack a day for past 30 years
. Alcohol : Occasional beer
. Recreational drugs : No

Physical examination

Abdomen:
. No bruits

Extremities
. Pulse 2+ and symmetrical in bilateral lower extremities

Musculoskeletal
. Negative Homans sign
. ,No calf tenderness to palpation bilaterally

Neurologic
. Motor strength 5/5 in both lower extremities
. Grossly intact sensation
. Deep tendon reflexes 2+ symmetrically

¡X¡X¡X¡X¡X

Case 20 sim. pt. checklist

Following the encounter , check which of the following items were performed by the examinee

History of present illness/ review of systems

. Asked about the location and radiation of the pain
. Asked about the onset of pain
. Asked about whether it is continuous or intermittent pain
. Asked about the intensity of pain
. Asked about the quality of pain
. Asked about the course of pain over time
. Asked about any aggravating or relieving factors
. Asked about rest pain
. Asked about swelling of the legs
. Asked about sensory changes ( such as numbness ) and paresthesia
. Asked boy any weakness of the legs
. Asked about any history of back pain
. Asked about fever
. Asked about trauma to the legs
. Asked about other joint pain
. Asked about recent surgeries or prolonged immobilization
. Asked about impotence

Past medical /family/social history

. Asked about similar episodes in the past
. Asked about past medical issues (especially high blood pressure , diabetes , high cholesterol, disc prolapse)
. Asked about current medications
. Asked about family health (especially history of blood clots)
. Asked about tobacco , alcohol , and recreational drug use
. asked bout occupation

Examination

. Washed heads before examination
. Examined without gown , not through gown
. Examined calf tenderness
. Elicited Homans sign
. Checked pulses in both legs and  arms
. Listened for bruits at the distal aorta , iliac , or femoral arteries
. Checked sensation in both legs
. Checked reflexes in bother legs
. Checked for vibration sense in both legs

Counseling

. Explained physical findings and possible diagnosis
. Explained further workup
. Discussed lifestyle modifications, including quitting smoking

Communication skills and professional conduct

. Knocked before entering the room
. Introduced self and greeted you warmly
. Used your name to address you
. Paid attention to what you said and maintained good eye contact
. Asked opened questions
. Asked non-leading questions . Asked one question at a time
. Listened to what you said without interrupting . Used plain English rather than technical jargon
. Used appropriate transition sentences
. Used appropriate draping techniques
. Summarized the history and explained physical findings
. Expressed empathy and gave appropriate reassurances
. Asked whether you have any concerns/questions

Differential diagnosis

. Thromboangiitis obliterans
. Atherosclerotic vascular disease
. Drug induced (statins)
. Diabetic polyneuropathy

Diagnostic study/studies

. Creatinine kinase
. Blood sugar and hemoglobin A1c
. Lipid profile
. Arterial doppler study coif the lower extremities
. Duplex venous ultrasound of power limbs
. CBC with differential
. Spine MRI

¡X¡X¡X¡X¡X

Case 20 clinical summary

Clinical Skills Evaluation
Case 20 Patient Note

The following represents a typical note for this patient encounter . the detail may vary depending on the information given by the simulated patient.

History : Describe the history you just obtained form this patient . Include only information (Pertinent positives and negatives)  relevant to this patient¡¦s problem(s).

. 50-yo man with 2 months of worsening bilateral leg pain
. Trolling pain mainly over the calf muscle made worse with walking , running, and prolonged standing.
. Symptom improvement with rest and sitting

ROS : No pain at rest , fever , trauma , swelling , back pain, weakness , sexual difficulties , numbness , or tingling in legs
PMHx : Diabetes for 3 years under diet control , hypercholesterolemia
PSHx : None
Meds: Simvasatin 40 mg daily at bedtime
Allergies : None
FHx : Father died at age 65 of stroke ; mother and 2 siblings are healthy
SHx : 2 PPD smoker for past 30 years , occasional alcohol use

Physical examinations : Describe any positive and  negative findings relevant  to this patient¡¦s problem(s) . be careful to include only those parts of the examination performed in this encounter.

. Vital signs : Temperature , 36.7¡¦C (98.1F); Blood pressure , 140/80 mmHg; pulse , 78/min; and respirations ,20/min
. Abdomen : No bruits
. Extremities : Pulse 2+ and symmetrical in bilateral lower extremities
. Musculoskeletal : Negative Homans sign , no calf tenderness to palpation bilaterally
. Neurologic : bilateral lower extremities with 5/5 motor strength , intact vibratory sensation and proprioception, and DTR 2+

Data interpretation : Based on what you have learned from the history and physical examination, list up to 3 diagnosis that might explain this patient¡¦s complaint(s) . List your diagnoses from most to least likely . For some cases , fewer than 3 diagnosis will be appropriate . the , enter the positive and negative findings form the history and the physical examination (if present) that support each diagnosis . Finally , list initial diagnostic (if any) you would order for each listed diagnosis (eg,restricted physical examination maneuvers, laboratory tests , imaging ECG , etc).

Diagnosis #1 : Drug-induced (statin) myopathy

History finding(s)
. Proximal muscle pain
. History of statin use

Physical examination finding(s)
. Normal motor strength in legs
. No sensory deficits in legs

Diagnosis #2 : Atherosclerotic peripheral vascular disease

History finding(s)
. History of diabetes and high cholesterol
. History of smoking
. Family history of stroke

Physical examination finding(s)
. None

Diagnosis #3 : Diabetic polyneuropathy

History finding(s)
. Diet-controlled diabetes
. History of high cholesterol

Physical examination finding(s)
. None

Diagnostic studies
. Creatinine kinase
. Fasting blood sugar and hemoglobin A1c
. Lower-extremity arterial Doppler







µoªí¤å³¹®É¶¡2018/08/15 03:37pm¡@IP: ¤w³]©w«O±K[¥»¤å¦@ 8262 ¦ì¤¸²Õ]¡@ 
JuanFe 

 

¸ê®Æ: ¦¹·|­û¥Ø«e¤£¦b½u¤W Male ¨°Às ¥Õ¦Ï®y
«Â±æ: 0
¾y¤O: ¾y¤O: 78957
¸gÅç: ¸gÅç: 35986
¨Ó¦Û: ¦t©z¤¤¡@blank
µo¤å: 1118 ½g
ºëµØ: 0 ½g
¦b½u: 47¤Ñ19®É22¤À57¬í
µù¥U: 2013/06/17
Message ¬d¬Ý¡@·j´M¡@³q°T¿ý¡@½Æ»s¡@¤Þ¥Î¡@¦^ÂФ峹¦^ÂС@[²Ä 22 ¼Ó]
 21 Case 21 scenario (vomiting)

Doorway information about patient

The patient is a 56-year-old woman who comes to the emergency department due to vomiting

Vital signs

. Temperature : 36.7¡¦C (98F)
. Blood pressure : 90/60 mmHg
. Pulse : 98/min
. Respirations : 20/min

Clinical Images

The paint has vomited into a pan of water at the bedside , as shown in the image : frank-blood or coffee-ground vomiting

Basic differential diagnosis

. Peptic ulcer disease
. Gastric erosion
. Esophageal varices
. Mallory-Weiss tears
. Esophagitis
. Duodenitis
. Malignancy (esophageal and gastric)

¡X¡X¡X¡X¡X

Case 21 sim.pt. instruction

If the doctor asks you about anything other than these , just say ¡§ no ,¡¨ or provide an answer that a normal patient might give.

You are a 56-year-old woman who is vomiting up blood

History of present illness

. Felt lightheaded while climbing stairs at home and passed out (2 hours ago)
. Sharp mid-epigastric pain starting 1 day ago (4-5/10 severity , no radiation)
. Nausea that is worse today
. Threw up a teaspoon of blood twice at home ; threw up more blood in the emergency department
. Back stools for 1 week

Review of systems

. No changes in appetite or weight
. No fever or chills
. No shortness of breath
. No otters dizziness or chest pain

Past medical history / family / social history

. GERD for past 2 years relieved with antacids as needed
. Chronic back pain
. No surgeries
. Medications : Ibuprofen as needed
. No allergies
. Father died of heart attack at age 60; mother and 2 siblings are healthy
. Single ,live with roommate . Tobacco : 1 pack a day for past 25 years
. Alcohol : 4-5 beers a day for past 20 years
. Recreational drugs : No

Do not volunteer this information unless asked about problems with drinking: You were admitted to an alcohol; treatment facility 1 years ago and left after 1 week

Physical examination

HEENT :
. Oropharynx clear

CV :
. Regular rate and rhythm
. No murmurs

Abdomen :
. Non-tender , non-distended
. Normative bowel sounds
. No hepatosplenomegaly

¡X¡X¡X¡X¡X

Case 21 sim. pt. checklist

Following the encounter , check which of the following items were performed by the examinee

History of present illness/ review of systems

. Asked about the onset of vomiting
. Asked about the frequency of vomiting
. Asked about any blood in the vomit (frank-blood or coffee-ground vomiting) and quantity of blood
. Asked about any abdomen pain associated with the vomiting
. Asked about prior history abdomen pain ( or heartburn ) especially in relation to food . Asked specifically about melena (black stools)
. Asked about recent change in appetite and weight loss

Past medical /family/social history

. Asked about similar episodes in the past
. Asked about other medical issues (peptic ulcer disease , reflux disease , liver problems),hospitalizations , and surgeries
. Asked about current medications
. Asked about medication allergies
. Asked about family health (including bleeding problems)
. Asked about tobacco , alcohol , and recreational drug use (including detailed discussion of alcohol abuse and treatment)
. Asked about occupation

Examination

. Washed heads before examination
. Examined without gown , not through gown
. Performed orthostatic vital signs
. Examined oropharynx
. Auscultated abdomen (prior to palpation)
. Palpated abdomen (superficial and deep)
. Checked for rigidity and rebound
. Percussed for liver span
. Performed neurologic examination
. Performed cardiovascular examination

Counseling

. Explained physical findings and possible diagnosis
. Explained further workup (blood test,endoscopy)
. Explained the importance of lifestyle modifications , including quitting smoking and alcohol

Communication skills and professional conduct

. Knocked before entering the room
. Introduced self and greeted you warmly
. Used your name to address you
. Paid attention to what you said and maintained good eye contact
. Asked opened questions
. Asked non-leading questions . Asked one question at a time
. Listened to what you said without interrupting me
. Used plain English rather than technical jargon
. Used appropriate transition sentences
. Used appropriate draping techniques
. Summarized the history and explained physical findings
. Expressed empathy and gave appropriate reassurances
. Asked whether you have any concerns/questions

Differential diagnosis

. Variceal hemorrhage
. Peptic ulcer decease
. Gastric erosions
. Esophagitis
. Duodenitis

Diagnostic study/studies

. CBC with differential count
. PT/PTT/INR
. BUN, serum creatinine , electrolytes
. Upper GI endoscopy
. Liver function test (albumin, AST,ALT, alkaline phosphatase , total and direct bilirubin)
. ECG

¡X¡X¡X¡X¡X

Case 21 clinical summary

Clinical Skills Evaluation
Case 21 Patient Note

The following represents a typical note for this patient encounter . the detail may vary depending on the information given by the simulated patient.

History : Describe the history you just obtained form this patient . Include only information (Pertinent positives and negatives)  relevant to this patient¡¦s problem(s).

. 56-yo woman with syncope while climbing stairs
. Followed by recurrent hematemesis over the last 2 hours
. History of GERD  relieved with as-needed antacids.
. 1 day of mid-epigastric abdomen pain without radiation , associated with nausea
. 1 week of melena

ROS : No changes in appetite , weight loss , fever , chills , shortness of breath , or chest pain
PMHx: GERD , chronic back pain , alcohol abuse
PSHx: None
Meds : ibuprofen , as needed
Allergies : None
FHx: Father died at age 60 of heart attack ; motor and 2 siblings are healthy
SHx: 1 PPD smoker for past 20 years , 4-5 beers /day for past 20 years
Physical examinations : Describe any positive and  negative findings relevant  to this patient¡¦s problem(s) . be careful to include only those parts of the examination performed in this encounter.

. Vital signs : Temperature ,36.7¡¦C(98.1F); blood pressure , 90/60 mmHg; pulse , 98/min ; and respirations , 20/min
. HEENT : Oropharynx clear
. Heart : RRR with no M,G,R. Abdomen : non-tender , non-distended , normative bowel sounds , no hepatosplenomegaly

Data interpretation : Based on what you have learned from the history and physical examination, list up to 3 diagnosis that might explain this patient¡¦s complaint(s) . List your diagnoses from most to least likely . For some cases , fewer than 3 diagnosis will be appropriate . the , enter the positive and negative findings form the history and the physical examination (if present) that support each diagnosis . Finally , list initial diagnostic (if any) you would order for each listed diagnosis (eg,restricted physical examination maneuvers, laboratory tests , imaging ECG , etc).

Diagnosis #1 : Bleeding esophageal varices

History finding(s)
. History of ibuprofen and excessive alcohol use
. Mid-epigastric pain
. Hematemesis and melena

Physical examination finding(s)
. None

Diagnosis #2 : Bleeding peptic ulcer

History finding(s)
. History of GERD and excessive alcohol use
. Mid-epigastric pain and hematemesis
. Melena

Physical examination finding(s)
. None

Diagnosis #3 : Gastritis

History finding(s)
. History of ibuprofen and excessive alcohol use
. Mid-epigastric pain
. Melena

Physical examination finding(s)
. None

Diagnostic studies

. Orthostatic BP and HR measurement . CBC with differential . Basic metabolic panel
. Liver function test . PT/PTT/INR
. Upper GI endoscopy








µoªí¤å³¹®É¶¡2018/08/15 03:38pm¡@IP: ¤w³]©w«O±K[¥»¤å¦@ 7952 ¦ì¤¸²Õ]¡@ 
JuanFe 

 

¸ê®Æ: ¦¹·|­û¥Ø«e¤£¦b½u¤W Male ¨°Às ¥Õ¦Ï®y
«Â±æ: 0
¾y¤O: ¾y¤O: 78957
¸gÅç: ¸gÅç: 35986
¨Ó¦Û: ¦t©z¤¤¡@blank
µo¤å: 1118 ½g
ºëµØ: 0 ½g
¦b½u: 47¤Ñ19®É22¤À57¬í
µù¥U: 2013/06/17
Message ¬d¬Ý¡@·j´M¡@³q°T¿ý¡@½Æ»s¡@¤Þ¥Î¡@¦^ÂФ峹¦^ÂС@[²Ä 23 ¼Ó]
 22 Case 22 scenario (chest pain)

Doorway information about patient

The patient is a 55-year-old man who comes to the emergency department due to chest pain

Vital signs

. Temperature : 37.1 C (98.7F)
. Blood pressure : 130/80 mmHg
. Pulse : 78 /min
. Respirations : 20/min

Clinical images

ECG is shown in the image : S-T segment lowered

Basic differential diagnosis

. Miocardio infarction
. Unstable angina
. Pulmonary embolism
. Costochrondritis
. Pleuritis
. Pericarditis
. Aortic dissection
. Gastroesophageal reflux
. Esophageal; perforation

¡X¡X¡X¡X¡X

Case 22 sim.pt. instruction

If the doctor asks you about anything other than these , just say ¡§ no ,¡¨ or provide an answer that a normal patient might give.

You are a 55-year-old man who comes to the emergency department with chest pain.

History of present illness

. The pain came on suddenly and has progressively worsened
. Pain located in substernal area with no radiation
. ¡§Tight , squeezing ¡§ sensation with 8-9/10 severity
. Pain is worse when walking and moving around
. Associated symptoms:
- Nausea
- 1episode of vomiting
- Sweating
- Mild shortness of breath

Review of systems

. No fever , cough , headache . abdominal pain , diarrhea , constipation , recent trauma, appetite changes   weight loss , or urinary problems Past medical history

. High blood pressure for 20 years
. Diabetes for 5 years
. Cholesterol tested a year ago was 280 ( you are trying to control your cholesterol who diet but not eat a lot of fast food)
. No surgires
. Medications : lisinopril , metformin
. No allergies
. Father died at age 60 of heat attack ; mother tis living and ad stroke at age 65 ; brother had a heart attack at age 58
. Occupation : lawyer
. Married , live with wife
. Tobacco : 1 pack a day for the past 30 years
. Alcohol : 1 glass of wine a day for past 20 years
. Recreational drugs : No

Physical examination

Physical examination

Neck : . supple without JVD or lymphadenopathy
. No thyromgaly

Lungs :
. Clear to auscultation bilaterally
. No reproducible chest pain with palpation

Heart :
. Regular rate and rhythm
. No murmurs , gallops , or rubs

Review of system

You have non of the following:
. Fever
. Cough
. Headache
. Abdominal pain
. Diarrhea
. Constipation
. Recent trauma
. Appetite changes
. Weight loss
. Urinary problems

¡X¡X¡X¡X¡X

Case 22 sim. pt. checklist

Following the encounter , check which of the following items were performed by the examinee

History of present illness/ review of systems

. Asked about the location and radiation of pain
. Asked about the intensity of pain
. Asked about the quality of pain
. Asked about the origin and duration of pain
. Asked about the course of pain over time
. Asked about any aggravating or relieving factors
. Asked about associated symptoms , especially :
- Nausea and vomiting
- Sweating
- Fever
- Coughing
- Shortness of breath
- Palpitations
- Syncope and dizziness

Past medical /family/social history

. Asked about similar episodes in the past
. Asked about past ,medical issue (especially high blood pressure , heart problems , diabetes , heart burn/reflux), hospitalizations , and surgeries
. Asked about current medications and medication allergies
. Asked about family health
. Asked about tobacco , alcohol , and recreational drug use
. Asked about occupation and stress in life
. Asked about cholesterol level (if known)

Examination

. Washed heads before examination
. Examined without gown , not through gown
. Examined carotid artery and jugular viens
. Examined heart (inspection , palpation , auscultation)
. Auscultated the lungs
. Examined peripheral pulse and edema
. Examined abdomen

Counseling

. Explained the physical findings and possible diagnosis
. Discussed ECG result
. Explained further workup
. Discussed lifestyle modifications ( especially quitting smoking and moderate alcohol intake).

Communication skills and professional conduct

. Knocked before entering the room
. Introduced self and greeted you warmly
. Used your name to address you
. Paid attention to what you said and maintained good eye contact
. Asked opened questions
. Asked non-leading questions . Asked one question at a time
. Listened to what you said without interrupting me
. Used plain English rather than technical jargon
. Used appropriate transition sentences
. Used appropriate draping techniques
. Summarized the history and explained physical findings
. Expressed empathy and gave appropriate reassurances
. Asked whether you have any concerns/questions

Differential diagnosis

. Myocardial infarction
. Unstable angina
. Pulmonary embolism
. Aortic dissection
. Gastroesphageal reflux

Diagnostic study/studies

. Complete blood count
. Cardiac markers (eg, troponin)
. Electrolytes . blood urea nitrogen, creating , glucose
. Chest x-ray
. Echocardiogram

¡X¡X¡X¡X¡X

Case 22 clinical summary

Clinical Skills Evaluation
Case  Patient Note

The following represents a typical note for this patient encounter . the detail may vary depending on the information given by the simulated patient.

History : Describe the history you just obtained form this patient . Include only information (Pertinent positives and negatives)  relevant to this patient¡¦s problem(s).

. 55-yo man with 2 hours of chest pain described as substernal tightness and pressure and increased  with movement and walking ; pain of 8-9 lb on a scale of 10 , no radiation.
. Associated nausea , vomiting , sweating , and shortness of breath

ROS : No fever , cough , headache , abdominal pain , diarrhea , constipation , recent trauma, appetite change , weight loss , or urinary problems
PMHx : HTN , diabetes , hight cholesterol
PSHx : None
Meds : lisinpril, metformin
Allergies : None
FHx: Fateghr died at age 60 of heat attack , motor had a stroke at age 65 , and mother had a heart attack at age 58
SHx: 1 PPF smoker for past 30 years , 1 glass of wine/day for past 20 years

Physical examinations : Describe any positive and  negative findings relevant  to this patient¡¦s problem(s) . be careful to include only those parts of the examination performed in this encounter.

. Vital signs ; Temperature ,. 37.1¡¨C (98.8F) : blood pressure , 130/80 mmHg; pulse , 78/min; and respirations , 20/min
. Neck ; Supple without JVD or lymphadenopathy , no thyromegaly
. Lungs ; Clear to auscultation bilaterally , no reproducible chest pain to palpation
. Heart : RRR without murmurs , gallops , or rubs

Data interpretation : Based on what you have learned from the history and physical examination, list up to 3 diagnosis that might explain this patient¡¦s complaint(s) . List your diagnoses from most to least likely . For some cases , fewer than 3 diagnosis will be appropriate . the , enter the positive and negative findings form the history and the physical examination (if present) that support each diagnosis . Finally , list initial diagnostic (if any) you would order for each listed diagnosis (eg,restricted physical examination maneuvers, laboratory tests , imaging ECG , etc).

Diagnosis #1 : Acute contrary syndrome

History finding(s)
. Substernal chest pain
. History of multiple cardiac risk factors
. Nausea , vomiting , diaphoreses

Physical examination finding(s)
. No reproducible chest pain to palpation

Diagnosis #2 : Aortic dissection

History finding(s)
. History of hypertension
. substernal pain
. Sudden-onset symptoms

Physical examination finding(s)
. No reproducible chest pain to palpation

Diagnosis #3 : Pulmonary embolism

History finding(s)
. Sudden -onset chest pain
. Shortness of breath

Physical examination finding(s)
. No reproducible chest pain to palpation

Diagnostic studies
. ECG shows ST depressions in V2-V5
. Chest x-ray
. Cardiac enzymes
. Echocardiogram







µoªí¤å³¹®É¶¡2018/09/13 07:49am¡@IP: ¤w³]©w«O±K[¥»¤å¦@ 8259 ¦ì¤¸²Õ]¡@ 
JuanFe 

 

¸ê®Æ: ¦¹·|­û¥Ø«e¤£¦b½u¤W Male ¨°Às ¥Õ¦Ï®y
«Â±æ: 0
¾y¤O: ¾y¤O: 78957
¸gÅç: ¸gÅç: 35986
¨Ó¦Û: ¦t©z¤¤¡@blank
µo¤å: 1118 ½g
ºëµØ: 0 ½g
¦b½u: 47¤Ñ19®É22¤À57¬í
µù¥U: 2013/06/17
Message ¬d¬Ý¡@·j´M¡@³q°T¿ý¡@½Æ»s¡@¤Þ¥Î¡@¦^ÂФ峹¦^ÂС@[²Ä 24 ¼Ó]
 23 Case 23 scenario ( frequent falls )

Doorway information about patient

The patient is a 70-year-old man who comes to the office due to frequent falls.

Vital signs
. Temperature : 37.1¡¦C (98.7F)
. Blood pressure : 130/80 mmHg
. Pulse : 78/min
. Respirations : 20/min

Basic differential diagnosis

Neurologic
. Cerebellar disease (alcohol, tumor , stroke)
. Parkinson disease
. Brain tumor
. Seizure
. Depressed vision

Metabolic
. Diabetic neuropathy
. Hypoglycemia
. Thyroid disease

Cardiovascular
. Valvular disease

Miscellaneous
. Medication side effect
. Vitamin B12 deficiency
. Vertigo

¡X¡X¡X¡X¡X

Case 23 sim.pt. instruction

If the doctor asks you about anything other than these , just say ¡§ no ,¡¨ or provide an answer that a normal patient might give.

You are a 70-year-old man who comes to the clinic due to frequent falls.

History of present illness

. Several falls over he last 2 months
. Initially once a week , now twice a week
. Decreased balance while standing
. No major injury/fracture but you are concerned that you may develop one
. Also have tremor that makes it difficult to hold things ; worse when reaching for an object
. Headache in the morning
. Friend said you speech is different

Review of systems

. No weakness , numbness , or tingling in arms or legs
. No dizziness/vertigo
. No fever
. No chest pain
. No nausea , vomiting , diarrhea , constipation , or abdominal pain
. No urinary symptoms
. No sin or hear changes

Past medical history

. Diabetes for last 10 years (under good control)
. No surgires
. Medications : Metformin 500mg twice a day
. No allergies
. Father and mother both died of¡¨old age¡¨; no siblings
. Retired machinist
. Widower (wife passed away 5 years ago), live alone
. Tobacco : No
. Alcohol : 2 beers a day for 40 years
. Recreational drugs : No

Physical examination

HEENT:
. Visual acuity and visual fields normal

Neck : . Supple without IVD or lymphadenopathy
. No thyromegaly
. No bruits

Lungs :
. Clear to auscultation bilaterally

Heart : . Regular rate and rhythm
. No murmurs , gallops , or rubs

Neurologic :
. Motor 5/5 bilaterally
. Sensory grossly intact bilaterally
. Resting tremor
. Mild dysmetria (finger to nose ) present
. Mild dysdiadochokinesia (alternating movements)
. DTR2+ bilaterally

¡X¡X¡X¡X¡X

Case 23 sim. pt. checklist

Following the encounter , check which of the following items were performed by the examinee

History of present illness/ review of systems

. Asked about the onset and frequent of falls
. Asked about any injury associated with falls
. Asked about loss of consciousness
. Asked about any difficult in initiating , controlling , stopping movements
. Asked about progression of the problem
. Asked about associated symptoms:
- Tremors
- Headache
- Nausea /vomiting, bowel problem
- Fever
- Palpations and syncope
- Thyroid symptoms(eg, temperature intolerance , skin or hear changes)
- Changes in appetite or weight
- Problems with speech or memory
- Problems wit attention or calculation
- urinary problem
. Asked abort living conditions and support systems

Past medical /family/social history

. Asked about similar episodes in the past
. Asked about past medical issues , hospitalizations , and surgeries
. Asked about current medications
. Asked about medication allergies
. Asked about family health
. Asked about tobacco, alcohol , and recreational drug use
. Asked about occupation

Examination

. Washed heads before examination
. Examined without gown , not through gown
. Checked orthostatic vital signs
. Examined eyes
. Examined heart and lungs
. Performed mini-mental status exam
. Examined touch , pain , and temperature sensations in legs add hands
. Tested muscle power in limbs
. Tested for muscle tone/rigidity
. Asked you to get up and walk and turn around and sit again (¡§ Get up and go ¡§ test)
. Performed finger nose test
. Performed alternating movements test
. Performed Romberg test
. Checked reflexes

Counseling

. Explained the physical findings and possible diagnosis
. Explained further workup

Communication skills and professional conduct

. Knocked before entering the room
. introduced self and greeted you warmly
. Used your name to address you
. Paid attention to what you said and maintained good eye contact
. Asked opened questions
. Asked non-leading questions . Asked one question at a time
. Listened to what you said without interrupting me
. Used plain English rather than technical jargon
. Used appropriate transition sentences
. Used appropriate draping techniques
. Summarized the history and explained physical findings
. Expressed empathy and gave appropriate reassurances
. Asked whether you have any concerns/questions

Differential diagnosis

. Cerebellar disease
. Parkinson disease
. Diabetic neuropathy
. Brain tumor
. Thyroid disease
. Vitamin B12 deficiency

Diagnostic study/studies

. CBC with differential . CT or MRI of brain
. Serum electrolytes, glucose, creatinine
. Hemoglobin A1c
. ECG
. TSH

¡X¡X¡X¡X¡X

Case 23 clinical summary

Clinical Skills Evaluation
Case 23 Patient Note

The following represents a typical note for this patient encounter . the detail may vary depending on the information given by the simulated patient.

History : Describe the history you just obtained form this patient . Include only information (Pertinent positives and negatives)  relevant to this patient¡¦s problem(s).

. 70-yo man with 2 months of frequent fall
. Balanced problems while standing up ; no syncope
. Tremor in hands worse when reaching for objects.
. Change in speech , occasional morning headache.
. No sensory symptoms (numbness , tingling) in legs.
. No dizziness or vertigo

ROS : No fever , nausea , hair loss, chest pin , abdominal , pain , recent trauma, diarrhea , constipation , or urinary problems
PMHx : Diabetes
PSHx : None
Meds : Metformin 500 mg BID
Allergies : None
FHx : Father and mother both fiddled of old age
SHx : No smoking , 2 beers daily for past 40 years

Physical examinations : Describe any positive and  negative findings relevant  to this patient¡¦s problem(s) . be careful to include only those parts of the examination performed in this encounter.

. Vital signs : Temperature , 37.1¡¦C (98.8F) ; blood pressure , 130/80 mmHg ; pulse , 78/min ; and respirations , 20 /min
. HEENT : PERRLA , EOMI , normla visual acuity
. Neck : Supple without JVD or lymphadenopathy , no thyromegaly , no bruits
. Lungs : Clear to auscultation bilaterally
. Heart : RRR without murmurs, gallops , and rubs
. Neurologic ; Motor 5/5 bilaterally , sensory grossly intact bilaterally , resting tremor , mild dysmetria (finger to nose) , mild dysdiadochokinesia (alternating movements) , DTR 2+ bilaterally

Data interpretation : Based on what you have learned from the history and physical examination, list up to 3 diagnosis that might explain this patient¡¦s complaint(s) . List your diagnoses from most to least likely . For some cases , fewer than 3 diagnosis will be appropriate . the , enter the positive and negative findings form the history and the physical examination (if present) that support each diagnosis . Finally , list initial diagnostic (if any) you would order for each listed diagnosis (eg,restricted physical examination maneuvers, laboratory tests , imaging ECG , etc).

Diagnosis #1 : Cerebellar disease due to alcohol use

History finding(s)
. History of chronic alcohol use
. Difficulty with balance

Physical examination finding(s)
. Dysmetria
. Disdiadochokineasia

Diagnosis #2 : Parkinson disease

History finding(s)
. Tremor
. Balance problems

Physical examination finding(s)
. Resting tremor

Diagnosis #3 : Brain tumor

History finding(s)
. Speech difficulties, headache
. Balance problems
. 2 months of symptoms

Physical examination finding(s)
. None

Diagnostic studies

. Orthostatic vitals
. Brain imaging (CT scan or MRI)
. Basic metabolic panel
. Thyroid function tests; vitamin B 12 levels
. Complete blood count







µoªí¤å³¹®É¶¡2018/09/13 07:50am¡@IP: ¤w³]©w«O±K[¥»¤å¦@ 8510 ¦ì¤¸²Õ]¡@ 
JuanFe 

 

¸ê®Æ: ¦¹·|­û¥Ø«e¤£¦b½u¤W Male ¨°Às ¥Õ¦Ï®y
«Â±æ: 0
¾y¤O: ¾y¤O: 78957
¸gÅç: ¸gÅç: 35986
¨Ó¦Û: ¦t©z¤¤¡@blank
µo¤å: 1118 ½g
ºëµØ: 0 ½g
¦b½u: 47¤Ñ19®É22¤À57¬í
µù¥U: 2013/06/17
Message ¬d¬Ý¡@·j´M¡@³q°T¿ý¡@½Æ»s¡@¤Þ¥Î¡@¦^ÂФ峹¦^ÂС@[²Ä 25 ¼Ó]
 24 Case 24 scenario ( cough and chest pain )

Doorway information about patient

The patient is a 35-year-old man who comes to the office due to cough and chest pain.

Vital signs

. Temperature L 38.7¡¦C (101.7F)
. Blood pressure : 130/80 mmHg
. Pulse ; 94/min
. Respirations : 24/min

Basic differential diagnosis

. Pneumonia . Pleuretic pain
. Pleural effusion
. Pulmonary edema
. Tuberculosis
. Pulmonary embolism
. Lung cancer
. Infective endocarditis
. GERD

¡X¡X¡X¡X¡X

Case 24 sim.pt. instruction

If the doctor asks you about anything other than these , just say ¡§ no ,¡¨ or provide an answer that a normal patient might give.

You are a 35-year-old man who comes to the clinic due to a cough

History of present illness

. Onset 2 days ago
. Started with ¡§ feeling tired and sick¡¨
. Productive cough with yellow sputum and blood streaks
. Sharp pain 5-6/10 severity at left chest ; worse with moving and any deep breath and better with exhalation
. Associated symptoms :
- Fever
- Chills
- Sweating
- Mild shortness of breath
. Exposure to ¡§ pneumonia ¡§ form a colleague at work

Ask the doctor : ¡§ Do I have pneumonia too?¡¨

Review of systems

. No changes in appetite
. No weight loss
. No abdominal pain
. No recent trauma
. No diarrhea or constipation
. No urinary symptoms

Past medical / family / social history

. Hospitalized once for chest pain 5 years ago with negative testing
. No surgieres
. No medications
. Allergies : Penicillin (rash)
. Father and mother are both healthy ; no siblings
. Occupations ; Investment advisor
. Tobacco : 1 pack a day for 15 years
. Alcohol : often go out with friends on weekends and drink average of 2 shot of liquor
. Recreational drugs : No

Physical examination

HEENT :
. Oropharynx clear

Neck :
. Supple without JVD and lymphadenopathy
. No thyromegaly
. No bruits
. No accessory muscle use

Lungs :
. Clear to auscultation bilaterally
. Fremitus symmetrical bilaterally
. Resonant to percussion bilaterally
. No bronchophony or egophony

Heart :
. Regular rate and rhythm
. No murmurs ,  gallops,  or rubs

Abdomen :
. Non-Tender, non-distended
. No hepatosplenomegaly
. Normative bowel sounds

¡X¡X¡X¡X¡X

Case 24 sim. pt. checklist

Following the encounter , check which of the following items were performed by the examinee

History of present illness/ review of systems

. Asked about the location and radiation of pain
. Asked about the intensity of pain
. Asked about the quality of pain
. Asked about the onset and duration of pain
. Asked about the course of pain one time
. Asked about any aggravating or relieving factors
. Asked about associated symptoms , especially :
- Vomiting
- Fever
- Coughing( and details of expectoration)
- Shortness of breath
- Hemoptysis
- Change in appetite

Past medical /family/social history

. Asked about similar episodes in the past
. Asked about past medical issues ( especially lung problems) , hospitalizations , and surgeries
. Asked about current medications
. Asked about medication allergies
. Asked about family health
. Asked about tobacco , alcohol , and recreational drug use
. Asked about sexual history
. Asked about occupation

Examination

. Washed heads before examination
. Examined without gown , not through gown¡¦
. Inspected neck , accessory muscles of respiration , and jugular views
. Examined heart : inspection , palpation , auscultation
. Examined lungs including:
- Inspection of lung inflation
- Anterior and posterior auscultation
- Percussion
- Tests for consolidation (tactile fremitus ,, egophony)
. Palpated abdomen for splenomegaly and hepatomegaly

Counseling

. Explained the physical findings and possible diagnosis
. Explained further workup
. Discussed quitting smoking

Communication skills and professional conduct

. Knocked before entering the room
. Introduced self and greeted you warmly
. Used your name to address you
. Paid attention to what you said and maintained good eye contact
. Asked opened questions
. Asked non-leading questions . Asked one question at a time
. Listened to what you said without interrupting me
. Used plain English rather than technical jargon
. Used appropriate transition sentences
. Used appropriate draping techniques
. Summarized the history and explained physical findings
. Expressed empathy and gave appropriate reassurances
. Asked whether you have any concerns/questions

Differential diagnosis

. Pneumonia
. Pleuritic pain
. Pleural effusion
. Pericarditis
. Lung cancer

Diagnostic study/studies

. CBC with differential count
. Sputum Gram stain, C/S
. ECG
. Chest x-ray
. Blood culture

¡X¡X¡X¡X¡X

Case 24 clinical summary

Clinical Skills Evaluation
Case 24 Patient Note

The following represents a typical note for this patient encounter . the detail may vary depending on the information given by the simulated patient.

History : Describe the history you just obtained form this patient . Include only information (Pertinent positives and negatives)  relevant to this patient¡¦s problem(s).

. 35-yo man with 2 days go product cough of yellow , blood-tinged sputum
. 2 dash pf sharp . left-sided chest pain worse with inspiration and improved with expiration.
. Fever . chills , sweating , and mild shortness of breath
. Sick contact in office

ROS : No changes in appetite our weight , abdominal pain, recent trauma , diarrhea , constipation , or urinary problems
PMHx : None
PSHx : None
Meds : None
Allergies ; Penicillin (rash)
FHx : Father and mother are healthy
SHx : 1 PPD smoker for past 15 tears , 2 shots a week for past 10 years

Physical examinations : Describe any positive and  negative findings relevant  to this patient¡¦s problem(s) . be careful to include only those parts of the examination performed in this encounter.

. Vital signs: Temperature , 38.7¡¦C(101.7F) ; blood pressure , 130/80 mmHg ; pulse , 94/min ; and respirations , 24/min
. HEENT ; Oropharynx clear
. Neck : Supple without JVD or lymphadenopathy , no thyromegaly, no bruits , no accessory muscle use
. Lungs ; clear to auscultation bilaterally , fremitus symmetrical bilaterally , resonant to percussion bilaterally , no brochophony or egophony
. Heart : RRR without murmurs, gallops, our rubs
. Abdomen : Non-tender , non-distended , no hepatopslenomegaly , normative bowel sounds

Data interpretation : Based on what you have learned from the history and physical examination, list up to 3 diagnosis that might explain this patient¡¦s complaint(s) . List your diagnoses from most to least likely . For some cases , fewer than 3 diagnosis will be appropriate . the , enter the positive and negative findings form the history and the physical examination (if present) that support each diagnosis . Finally , list initial diagnostic (if any) you would order for each listed diagnosis (eg,restricted physical examination maneuvers, laboratory tests , imaging ECG , etc).

Diagnosis #1 : Acute bronchitis

History finding(s)
. Smoking history
. Fever
. Cough productive of yellow , blood-tinged sputum

Physical examination finding(s)
. Fever

Diagnosis #2 : Pneumonia

History finding(s)
. Fever and chills
. Cough productive of yellow, blood-tinged sputum
. Pleuritic chest pain
. Sick contact at office

Physical examination finding(s)
. Fever
. Respirations , 24/min

Diagnosis #3 : Lung cancer

History finding(s)
. Smoking history
. Cough productive of blood-tinged sputum

Physical examination finding(s)
. None

Diagnostic studies
. Chest x-ray
. Sputum Gram stain and culture
. CBC with differential







µoªí¤å³¹®É¶¡2018/09/13 07:51am¡@IP: ¤w³]©w«O±K[¥»¤å¦@ 8032 ¦ì¤¸²Õ]¡@ 
JuanFe 

 

¸ê®Æ: ¦¹·|­û¥Ø«e¤£¦b½u¤W Male ¨°Às ¥Õ¦Ï®y
«Â±æ: 0
¾y¤O: ¾y¤O: 78957
¸gÅç: ¸gÅç: 35986
¨Ó¦Û: ¦t©z¤¤¡@blank
µo¤å: 1118 ½g
ºëµØ: 0 ½g
¦b½u: 47¤Ñ19®É22¤À57¬í
µù¥U: 2013/06/17
Message ¬d¬Ý¡@·j´M¡@³q°T¿ý¡@½Æ»s¡@¤Þ¥Î¡@¦^ÂФ峹¦^ÂС@[²Ä 26 ¼Ó]
 25 Case 25 scenario ( lower abdominal pain )

Doorway information about patient

The parents is 60-year-old man who comes to the emergency department due to lower abdominal pain

Vital signs
. Temperature : 38.3¡¦C (101F)
. Blood pressure : 130/84 mmHg
. Pulse : 98/min
. Respirations : 22/min

Basic differential diagnosis

. Diverticulitis
. Renal colic
. Appendicitis
. Ischemic colitis
. Infectious colitis
. Abdominal aortic aneurysm
. Intestinal obstruction

¡X¡X¡X¡X¡X

Case 25 sim.pt. instruction

If the doctor asks you about anything other than these , just say ¡§ no ,¡¨ or provide an answer that a normal patient might give.

You are a 60-year-old man who comes to the emergency department with lower abdominal pain

History of present illness

. The pain onset was 1 day ago after eating at a restaurant with family
. Pain is located at lower abdomen with no radiation
. 6/10 severity , progressively with no radiation
. Episodic (10-15 minute episodes ) . crampy pain
. No aggravated or alleviating factors
. Associated symptoms
- 1 episode non bilious , non bloody vomiting
- You did not check temperature but feel ¡§ a little feverish¡¦
- 2-3 episodes of diarrhea with visible blood (no black stools)
. No family members with symptoms
. No recent travel or sick contact

Do not volunteer this information unless asked :

Review of systems

. No chills
. No urinary symptoms

Past medical / family / social history

. Hospitalized once 10 years ago for kidney stone that passed spontaneously
. Hypertension
. No surgeries
. Medications : Hydrochlorothiazide 25 mg daily
. No allergies
. Father died at 65 of colon cancer ; mother died at 70 of breath cancer ; no siblings
. Occupation : Financial planner
. Married , live with wife
. Tobacco : No
. Alcohol : 1 beer a day for past 30 years
. Recreational drugs : No

Physical examination

Heart :
. Regular rate and rhythm
. No murmurs , gallops , or rubs

Abdomen :
. LLQ tenderness to deep palpation , no rebound tenderness
. Non-distended
. No hepatosplenomegaly
. Normative bowel sounds
. No CVA tenderness

¡X¡X¡X¡X¡X

Case 25 sim. pt. checklist

Following the encounter , check which of the following items were performed by the examinee

History of present illness/ review of systems

. Asked about the location and radiation of pain
. Asked about the intensity of pain
. Asked about the quality of pain
. Asked about the onset and duration of pain
. Asked about the progression of pain
. Asked about any aggravating or relieving factors
. Asked about associated symptoms , especially :
- Nausea and vomiting
- Fever and chills
- Changes in appetite and weight
- Bowel problems (constipation and diarrhea)
- Blood in the stool or black stood
- Urinary symptoms
. Asked bout recent travel and contaminated food ingestion
. Asked about recent antibiotic use

Past medical /family/social history

. Asked about similar episodes in the past
. Asked about past medical issues , hospitalizations ,ad surgeries (especially abdominal surgeries)
. Asked about current medications
. Asked about medication allergies
. Asked about family health
. Asked about tobacco , alcohol , and recreational drug use

Examination

. Washed heads before examination
. Examined without gown , not through gown
. Auscultated abdomen
. Palpated abdomen
. Checked rebound tenderness
. Checked for costovertebral angel tenderness
. Examined the heart

Counseling

. Explained physical findings and possible diagnosis
. Explained further workup
. Discussed need to perform rectal examination

Communication skills and professional conduct

. Knocked before entering the room
. Introduced self and greeted you warmly
. Used your name to address you
. Paid attention to what you said and maintained good eye contact
. Asked opened questions
. Asked non-leading questions . Asked one question at a time
. Listened to what you said without interrupting me
. Used plain English rather than technical jargon
. Used appropriate transition sentences
. Used appropriate draping techniques
. Summarized the history and explained physical findings
. Expressed empathy and gave appropriate reassurances
. Asked whether you have any concerns/questions

Differential diagnosis

. Diverticulitis
. Renal colic
. Intestinal obstruction
. Infectious colitis
. Ischemic colitis

Diagnostic study/studies

. Rectal examination , genital examination
. CBC with differential count
. Electrolytes , glucose , BUN , creatinine
. Urinalysis
. ECG
. Abdomen x-ray
. CT scan of the abdomen and pelvis

¡X¡X¡X¡X¡X

Case 25 clinical summary

Clinical Skills Evaluation
Case 25 Patient Note

The following represents a typical note for this patient encounter . the detail may vary depending on the information given by the simulated patient.

History : Describe the history you just obtained form this patient . Include only information (Pertinent positives and negatives)  relevant to this patient¡¦s problem(s).

. 60-yo man with 1 day of episodic , crapmy LLQ abdominal pain lasting 10-15 minutes each time ; no radiation.
. Pian is 6/10
. Ate at a local restaurant with family , but no one else is sick
. Nausea ,  1 episode of vomiting , 2-3 episodes of diarrhea with blood , subjective fever

ROS :  no recent travel , sick contact , chills , or urinary problems
PMHx : HTN , kidney stone in past
PSHx : None
Meds : Hydrochlorothiazide 25 mg daily
Allergies ; None
FHx : Father died at age 65 of colon cancer ; mother died at age 70 of breast cancer
SHx : No smoking , 1 beer daily for past 30 years

Physical examinations : Describe any positive and  negative findings relevant  to this patient¡¦s problem(s) . be careful to include only those parts of the examination performed in this encounter.

. Vital signs : Temperature , 38.3¡¦C (100.9F) ; blood pressure , 130/84 mmHg; pulse , 98/min; and respirations , 22/min
. Heart : RRR without murmurs , gallops , or rubs
. Abdomen : LLQ tenderness to deep palpation , no rebound tenderness , non distended , no hepatosplenomegaly , normative bowel sounds , no CVA tenderness

Data interpretation : Based on what you have learned from the history and physical examination, list up to 3 diagnosis that might explain this patient¡¦s complaint(s) . List your diagnoses from most to least likely . For some cases , fewer than 3 diagnosis will be appropriate . the , enter the positive and negative findings form the history and the physical examination (if present) that support each diagnosis . Finally , list initial diagnostic (if any) you would order for each listed diagnosis (eg,restricted physical examination maneuvers, laboratory tests , imaging ECG , etc).

Diagnosis #1 : Acute diverticulitis

History finding(s)
. Abdominal pain
. Fever
. Diarrhea with blood

Physical examination finding(s)
. Fever
. LLQ abdomen tender to palpation

Diagnosis #2 : Infectious colitis

History finding(s)
. Fever
. Ate at local restaurant before symptom onset
. Diarrhea with blood

Physical examination finding(s)
. Fever
. Abdomen tender to palpation

Diagnosis #3 : Ischemic colitis

History finding(s)
. History of hypertension
. Fever
. Abdominal pain

Physical examination finding(s)
. Fever

Diagnostic studies
. Rectal examination with stool guaiac
. Abdominal imaging (x-ray, CT scan)
. CBC with differential
. Urinalysis







µoªí¤å³¹®É¶¡2018/09/13 07:52am¡@IP: ¤w³]©w«O±K[¥»¤å¦@ 7770 ¦ì¤¸²Õ]¡@ 
JuanFe 

 

¸ê®Æ: ¦¹·|­û¥Ø«e¤£¦b½u¤W Male ¨°Às ¥Õ¦Ï®y
«Â±æ: 0
¾y¤O: ¾y¤O: 78957
¸gÅç: ¸gÅç: 35986
¨Ó¦Û: ¦t©z¤¤¡@blank
µo¤å: 1118 ½g
ºëµØ: 0 ½g
¦b½u: 47¤Ñ19®É22¤À57¬í
µù¥U: 2013/06/17
Message ¬d¬Ý¡@·j´M¡@³q°T¿ý¡@½Æ»s¡@¤Þ¥Î¡@¦^ÂФ峹¦^ÂС@[²Ä 27 ¼Ó]
 26 Case 26 scenario ( fatigue )

Doorway information about patient

The patient is a 35-year-old man who comes to the office due to fatigue

Vital signs

. Temperature : 37¡¦C (98.6F)
. Blood pressure : 120/80 mmHg
. Pulse : 82/min
. Respirations : 16/min

Basic differential diagnosis

. Depression
. Anemia
. Thyroid disorder
. Chronic fatigue syndrome

¡X¡X¡X¡X¡X

Case 26 sim.pt. instruction

If the doctor asks you about anything other than these , just say ¡§ no ,¡¨ or provide an answer that a normal patient might give.

You are a 35-year-old man who comes to the office due to fatigue

History of present illness

. Onset 2 months ago
. Previously well until victim of robbery
. Tired during the day with constant anxiety and impaired concentration
. Difficulty falling asleep at night with frequent nightmare
. You have cut back on daily activities and feel emotionally distant and lonely
. No other significant stress at work or home

Do not volunteer this information unless asked :

Review of systems

. No shortness of breath
. No palpations . No seating , fever , or chills
. No weight loss
. No change in appetite

Past medical / family / social history

. No significant illness , surgeries , or hospitalizations
. No medications . No allergies
. Immediate family members are all healthy
. Live with girlfriend
. Occupation : Florist
. Tobacco : 1 pack a day for last 15 years
. Alcohol : Social occasions only
. Recreational drugs : No

Physical examination

HEENT . No pallor
. Oropharynx clear

Neck
. Supple without lymphadenopathy
. No thyromegaly

Heart
. Regular rate and rhythm
. No murmurs, gallops, or rubs

Lungs :
. Clear to auscultation

Abdomen :
. No masses or tenderness

Psychiatric
. Alert and oriented to person , place , and time

¡X¡X¡X¡X¡X

Case 26 sim. pt. checklist

Following the encounter , check which of the following items were performed by the examinee

History of present illness/ review of systems

. Asked about onset of symptoms
. Asked about severity and change over time
. Asked if you are having difficulty falling or staying asleep
. Asked nightmares
. Asked if you had any traumatic events recently
. Asked about feeling of guilt
. Asked about suicidal intentions
. Asked if you have been feeling lonely
. Asked about anxiety
. Asked about associated symptoms , especially
- Palpitations
- Dizziness
- Sweating
- Tremors
- Changes in appetite or weight
- Shortness of breath
- Swelling /limps in neck
- Changes in bowel or bladder habits
. Asked about stress at work or home

Past medical /family/social history

. Asked about similar episodes in the past
. Asked about past medical issues , hospitalizations , and surgeries
. Asked about current medications
. Asked about medication allergies
. Asked about family health
. Asked about occupation
. Asked bout tobacco , alcohol , and recreational; drug use
. Asked about sexual history

Examination

. Washed heads before examination
. Examined without gown , not through gown
. Examined oral mucous membranes for pallor
. Palpated neck for masses or swelling
. Checked memory , orientation , and judgement

Counseling

. Explained physical findings and possible diagnosis
. Explained further workup
. Offered to help and support while getting treated
. Discussed the importance of quitting smoking and offered help

Communication skills and professional conduct

. Knocked before entering the room
. Introduced self and greeted you warmly
. Used your name to address you
. Paid attention to what you said and maintained good eye contact
. Asked opened questions
. Asked non-leading questions . Asked one question at a time
. Listened to what you said without interrupting me
. Used plain English rather than technical jargon
. Used appropriate transition sentences
. Used appropriate draping techniques
. Summarized the history and explained physical findings
. Expressed empathy and gave appropriate reassurances
. Asked whether you have any concerns/questions

Differential diagnosis

. Post-Traumatic stress disorder/anxiety disorder
. Depression
. Hypothyroidism
. Occult medical disease

Diagnostic study/studies

. CBC with differential
. TSH
. Electrolytes , glucose , BUN , Creatinine
. HIV test

¡X¡X¡X¡X¡X

Case 26 clinical summary

Clinical Skills Evaluation
Case 26 Patient Note

The following represents a typical note for this patient encounter . the detail may vary depending on the information given by the simulated patient.

History : Describe the history you just obtained form this patient . Include only information (Pertinent positives and negatives)  relevant to this patient¡¦s problem(s).

. 35-yo man with 2 months of fatigue after robbery
. Insomnia increased daytime fatigue , nightmares every night , generalized anxiety throughout the day , and inability to concentrate at work
. Feels emotionally alone and distant , no increased stress at work or home
. No hallucinations or delusions.
. Constipation for 3-4 months

ROS : No shortness of breath , chest pain , palpations , sweating, fever , chills , weight loss, or change in appetite
PMHx : None PSHx : None
Meds : None
Allergies : None
FHx : Father , mother , and 3 siblings are healthy
SHx : 1 PPD smoker for 15 years , occasional alcohol use

Physical examinations : Describe any positive and  negative findings relevant  to this patient¡¦s problem(s) . be careful to include only those parts of the examination performed in this encounter.

. Vital signs: Temperature , 36¡¦C (98.6F) ; blood pressure , 120/80 mmHg;pulse , 82/min; and respirations , 16/min
. HEENT : No pallor , oropharynx car
. Neck : Supple without lymphadenopathy or thyromegaly
. Heart : RRR without murmurs, gallops, or rubs
. Lungs ; Clear to auscultation
. Abdomen ; no masses or tenderness
. Psychiatric : Alert and oriented to person , place,and time

Data interpretation : Based on what you have learned from the history and physical examination, list up to 3 diagnosis that might explain this patient¡¦s complaint(s) . List your diagnoses from most to least likely . For some cases , fewer than 3 diagnosis will be appropriate . the , enter the positive and negative findings form the history and the physical examination (if present) that support each diagnosis . Finally , list initial diagnostic (if any) you would order for each listed diagnosis (eg,restricted physical examination maneuvers, laboratory tests , imaging ECG , etc).

Diagnosis #1 : Post-traumatic stress disorder

History finding(s)
. Symptom onset after robbery 2 months ago
. Insomnia
. Difficulty concentrating , nightmares

Physical examination finding(s)
. None

Diagnosis #2 : Depression

History finding(s)
. Fatigue
. Insomnia
. Feels alone and distant

Physical examination finding(s)
. None

Diagnosis #3 : Hypothyroidism

History finding(s)
. Constipation for 3-4 months
. Fatigue
. Inability to concentrate at work and home

Physical examination finding(s)
. None

Diagnostic studies

. TSH . CBC with differential
. Electrolytes, glucose, BUN , creatinine
. HIV test








µoªí¤å³¹®É¶¡2018/09/13 07:52am¡@IP: ¤w³]©w«O±K[¥»¤å¦@ 7490 ¦ì¤¸²Õ]¡@ 
JuanFe 

 

¸ê®Æ: ¦¹·|­û¥Ø«e¤£¦b½u¤W Male ¨°Às ¥Õ¦Ï®y
«Â±æ: 0
¾y¤O: ¾y¤O: 78957
¸gÅç: ¸gÅç: 35986
¨Ó¦Û: ¦t©z¤¤¡@blank
µo¤å: 1118 ½g
ºëµØ: 0 ½g
¦b½u: 47¤Ñ19®É22¤À57¬í
µù¥U: 2013/06/17
Message ¬d¬Ý¡@·j´M¡@³q°T¿ý¡@½Æ»s¡@¤Þ¥Î¡@¦^ÂФ峹¦^ÂС@[²Ä 28 ¼Ó]
 27 Case 27 scenario ( loss of hearing )

Doorway information about patient

The patient is a 65-year-old woman who comes to the office due to loss of hearing

Vital signs

. Temperature ; 36.7¡¦C (98.1F)
. Blood pressure : 130/86 mmhm
. Pulse ; 80 /min
. Respirations : 16/min

Basic differential diagnosis

conductive hearing loss
. Cerumen impaction
. Otitis media with effusion
. Tympanic membrane perforation
. Otosclerosis
. Foreign body in ear canal
. cholseteatoma
. Tympanosclerosis
. Tumor of the ear canal or middle ear

Sensorineural hearing loss
. Prescycusis (age-0related hearing loss)
. Ototoxicity
. Noise-induced hearing loss
. Meniere disease
. Diabetes
. Acoustic neuroma

¡X¡X¡X¡X¡X

Case 27 sim.pt. instruction

If the doctor asks you about anything other than these , just say ¡§ no ,¡¨ or provide an answer that a normal patient might give.

You are a 65-year-old woman who comes to the clinic due to hearing loss

History of present illness

. 3 months of reduced hearing in the left
. Hearing loss was initially mild but is progressively worsening
. Has been difficult to hear people with high-pitched voice
. No associated pain , ringing in the ear , or discharge from the ear
. Admitted to the hospital 4 months ago for a kidney infection and treated with IV antibiotics (you do not recall the name of the drug)

Review of systems

. No dizziness
. No facial muscle weakness
. No weakness or numbness i other parts of the body

Past medical / family / social history

. Hypertension for the last 30 years
. No surgires
. Medications : Hydrochlorothiazide 50 mg daily
. No drug allergies
. Father and mother both died of ¡¥ old age¡¨ ; 2 siblings , both healthy
. Occupations ; Supervisor at a steel factory (If the examinee specifically asks about the noise exposure , say ¡§ there is a lot of noise every day at work.¡¨)
. Married , live with husband
. Tobacco : No
. Alcohol : Occasional beer or wine
. Recreational drugs : No

Physical examination

HEENT :
. PERRLA , EOMI
. Oropharynx clear
. Tympanic membrane clear bilaterally
. Rinne test : Air conduction > bone conduction bilaterally
. Weber test ; localizes to the right ear

Neck:
. Supple
. No lymphadenopathy
. Thyroid normal

Neurologic: . Alert and oriental to person , place , and time
. Cranial nerves 2-12 intact , except for decreased hearing in the left ear

¡X¡X¡X¡X¡X

Case 27 sim. pt. checklist

Following the encounter , check which of the following items were performed by the examinee

History of present illness/ review of systems

. Asked which ear is affected (right , left , bilateral)
. Asked about the onset of symptoms
. Asked about the course of symptoms over time
. Asked about the subjective severity of hearing loss(mild , moderate , severe , profound)
. Asked about the possible initiating events
. Asked about associated symptoms , especially:
- Earache
- Tinnitus
- Vertigo
- Aural fullness
- Drainage from the ear
. Asked about any occupational exposure to noise
. Asked about trauma to the ear
. Asked about the social impact that it was had

Past medical /family/social history

. Asked about any similar episodes in the past
. Asked about past medical issues (especially ear. nose. throat and neurologic disorders ) ,hospitalizations , and surgeries
. Asked about current (and recent0 medications
. Asked about medication allergies
. Asked about family health
. Asked about tobacco , alcohol , and recreational drug use
. Asked about occupation

Examination

. Washed heads before examination
. Examined without gown , not through gown
. Examined eye movements and pupillary reflexes
. Examined facial sensation and motor function (eg , show teeth , puff out cheeks , stick out tongue)
. Examined external ear and ear canal ( with otoscope)
. Tested hearing , including Rinne and Weber tests

Counseling

. Explained physical findings and possible diagnosis
. Explained further workup

Communication skills and professional conduct

. Knocked before entering the room
. Introduced self and greeted you warmly
. Used your name to address you
. Paid attention to what you said and maintained good eye contact
. Asked opened questions
. Asked non-leading questions . Asked one question at a time
. Listened to what you said without interrupting me
. Used plain English rather than technical jargon
. Used appropriate transition sentences
. Used appropriate draping techniques
. Summarized the history and explained physical findings
. Expressed empathy and gave appropriate reassurances
. Asked whether you have any concerns/questions

Differential diagnosis

. Presbycusis
. Occupational exposure
. Ceremony impaction
. Drug induced

Diagnostic study/studies

. Serum electrolytes and blood sugar
. Audiometry
. MRI of the brain
¡X¡X¡X¡X¡X

Case 27 clinical summary

Clinical Skills Evaluation
Case 27 Patient Note

The following represents a typical note for this patient encounter . the detail may vary depending on the information given by the simulated patient.

History : Describe the history you just obtained form this patient . Include only information (Pertinent positives and negatives)  relevant to this patient¡¦s problem(s).

. 65-yo woman with 3 moths of heigh-frequency hearing loss
. Hospitalized for pyelonephritis 4 months ago and treated with unknown antibiotic.
. Exposure to loud noises at work in steel factory.

ROS : No discharge from ear , ringing in ear , dizziness , facial muscle weakness , weakness or numbness in other parts of the body , or earache
PMHx : HTN
PSHx : None
Meds : Hydrochlorothiazide 50 mg daily
Allergies : None
FHx : Father and mother died of old age , healthy siblings
SHx : No smoking , occasional alcohol use

Physical examinations : Describe any positive and  negative findings relevant  to this patient¡¦s problem(s) . be careful to include only those parts of the examination performed in this encounter.

. Vital signs : Temperature , 36.7¡¦C (98.1F) ; blood pressure , 130/86 mmHg; pulse , 80/min; and respirations , 16/min
. HEENT : PERRLA , EOMI , oropharynx clear , TMs clear bilaterally , Rinne test with AC>BC , Weber test localization to the right ear , oropharynx clear
. Neck l Supple without lymphadenopathy or thyromegaly
. Neurologic : Alert and oriented to person , place , and time ; cranial nerves II-XII intact except for decreased hearing in left ear

Data interpretation : Based on what you have learned from the history and physical examination, list up to 3 diagnosis that might explain this patient¡¦s complaint(s) . List your diagnoses from most to least likely . For some cases , fewer than 3 diagnosis will be appropriate . the , enter the positive and negative findings form the history and the physical examination (if present) that support each diagnosis . Finally , list initial diagnostic (if any) you would order for each listed diagnosis (eg,restricted physical examination maneuvers, laboratory tests , imaging ECG , etc).

Diagnosis #1 : Presbycusis (age-related hearing loss)

History finding(s)
. Decreased hearing for 2-3 months
. High-Frequency hearing loss

Physical examination finding(s)
. Weber test localization to right ear

Diagnosis #2 : Noise-induced hearing loss

History finding(s)
. Decreased hearing
. Work at still factory with loud noise

Physical examination finding(s)
. Weber test localization to right ear

Diagnosis #3 : Drug-induced hearing loss

History finding(s)
. History of recent antibiotic use

Physical examination finding(s)
. Weber test localization to right ear

Diagnostic studies
. Audiometric testing
. Electrolytes and blood sugar
. MRI of brain








µoªí¤å³¹®É¶¡2018/09/13 07:53am¡@IP: ¤w³]©w«O±K[¥»¤å¦@ 8060 ¦ì¤¸²Õ]¡@ 
JuanFe 

 

¸ê®Æ: ¦¹·|­û¥Ø«e¤£¦b½u¤W Male ¨°Às ¥Õ¦Ï®y
«Â±æ: 0
¾y¤O: ¾y¤O: 78957
¸gÅç: ¸gÅç: 35986
¨Ó¦Û: ¦t©z¤¤¡@blank
µo¤å: 1118 ½g
ºëµØ: 0 ½g
¦b½u: 47¤Ñ19®É22¤À57¬í
µù¥U: 2013/06/17
Message ¬d¬Ý¡@·j´M¡@³q°T¿ý¡@½Æ»s¡@¤Þ¥Î¡@¦^ÂФ峹¦^ÂС@[²Ä 29 ¼Ó]
 Case 28 scenario ( knee pain )

Doorway information about patient

The patient is a 53-year-old man who comes to the emergency department due to right knee pain and swelling.

Vital signs

. Temperature ; 38.3¡¦C (101F)
. Blood pressure : 130/60/mmHg
. Pulse : 80/min
. aspirations : 18/min

Basic differential diagnosis

. Osteoarthritis
. Septic arthritis and bursitis
. Pseudogout and gout
. Reactive arthritis
. Traumatic knee injury
. Lyme disease
. Monoarticular rheumatoid arthritis
. Psoriatic arthritis

¡X¡X¡X¡X¡X

Case 28 sim.pt. instruction

If the doctor asks you about anything other than these , just say ¡§ no ,¡¨ or provide an answer that a normal patient might give.

You are a 53-year-old man with right knee pain

History of present illness

. 2 days of worsening right knee pain
. Throbbing pain , 7/10 severity
. No radiation of pain
. no relief with ibuprofen
. No recent trauma
. Chronic bilateral knee pain starting a year ago that is worse with walking and thought to be due to being overweight
. Stiffness in multiple joints every morning for 10-15 minute that resolves spontaneously
. No other aggravating or relieving factors

Do not volunteer this information unless asked :

Review of systems

. No fever or chills
. No nausea , vomiting , diarrhea , or constipation
. No rash
. No recent travel or sick contacts
. No inset bite
. No urinary symptoms

Past medical / family / social history

. Hypertension for the past 10 years . No surgeries or hospitalizations
. Medications ; Hydrochlorothiazide 25 mg daily ; ibuprofen 600 mg up to 3 times a day as needed
. Medication allergies : None
. Father as hypertension and mother has pseudo gout ; no siblings
. Occupation : Librarian
. Married , live with wife
. Tobacco : none
. Alcohol ; 1 or 2 beers on social occasions
. Recreational drugs ; None

Physical examination

HEENT :
. PERRLA , EOMI
. No conjunctival abnormalities

Heart :
. Regular rate and rhythm
. No murmurs , gallops , or rubs

Musculoskeletal :
. Right knee is tender to palpation with decreased range of motion but no redness or warmth
. no other joint abnormalities

Skin :
. No rates or lesions

¡X¡X¡X¡X¡X

Case 28 sim. pt. checklist

Following the encounter , check which of the following items were performed by the examinee

History of present illness/ review of systems

. Asked about the onset of pain
. Asked bout the course of pain over time
. Asked about the intensity of pain
. Asked about the quality of pain
. Asked about the location of pain
. Asked about any radiation of pain
. Asked about any aggravating or relieving factors
. Asked about associated symptoms , especially :
- Fever
- Joint swelling
- Joint redness
- Rash
. Asked about history of trauma to the joint
. Asked about morning stiffness
. Asked about history of travel (especially areas with endemic Lyme disease)
. Asked about any rennet ticks bites
. Asked about any pain and swelling in the other joints
. Asked about nay recent history of febrile illness
. Asked about any eye symptoms

Past medical /family/social history

. Asked about similar episodes in the past or other joint problems
. Asked about past medical issue , surgeries , and hospitalizations
. Asked about current medications
. Asked about medication allergies
. Asked about family health (especially joint disorders)
. Asked bout tobacco , alcohol , and recreational drug use
. Asked about occupation
. Asked about living situation and sexual contacts

Examination

. Washed heads before examination
. Examined without gown , not through gown
. Checked knee for range of motion
. Checked other joints for swelling and redness
. Auscultated heart
. Examined eyes
. Examined skin for washed or painful nodules

Counseling

. Explained physical findings and possible diagnosis
. Explained further workup

Communication skills and professional conduct

. Knocked before entering the room
. introduced self and greeted you warmly
. Used your name to address you
. Paid attention to what you said and maintained good eye contact
. Asked opened questions
. Asked non-leading questions . Asked one question at a time
. Listened to what you said without interrupting me
. Used plain English rather than technical jargon
. Used appropriate transition sentences
. Used appropriate draping techniques
. Summarized the history and explained physical findings
. Expressed empathy and gave appropriate reassurances
. Asked whether you have any concerns/questions

Differential diagnosis

. Osteoarthritis
. Septic arthritis or bursitis
. Pseudo gout , gout
. Inflammatory (eg, rheumatoid ) arhritis

Diagnostic study/studies

. CBC with differential
. Joint aspiration
. X-ray of knee
. Sedimentation rate or C-reactive protein , antinuclear antibody, rheumatoid factor or cyclic citrullinated peptide antibodies
. MRI of joint
. Lyme serology (if travel to endemic area)

The following points should be addressed for traumatic knee pain:

. Asked what you were doing at the time of injury
. Asked about mechanism of injury
. Asked bout any noise or popping sensations at the time of injury
. Asked whether you can bear weight and whether the knee is unstable with walking
. Asked about locking of joint
. Performed Lachman maneuver or drawer test
. Performed McMurray maneuver

¡X¡X¡X¡X¡X

Case 28 clinical summary

Clinical Skills Evaluation
Case 28 Patient Note

The following represents a typical note for this patient encounter . the detail may vary depending on the information given by the simulated patient.

History : Describe the history you just obtained form this patient . Include only information (Pertinent positives and negatives)  relevant to this patient¡¦s problem(s).

. 53-yo man with 1 year of bilateral knee pain , now with 2 days of worsening right knee pain described as throbbing and constant
. Pain is 7/10 , no relief with ibuprofen
. 15-20 minute of morning stiffness in multiple joints each day that resolves spontaneously.
. No H/O trauma

ROS : No fever , chills , nausea , vomiting , diarrhea , constipation , rashes , travel history , sick contacts , insect bites , or urinary issue
PMHx : HTN for 10 years
PSHx : None
Meds : Hydrochlorothiazide 25mg  daily , ibuprofen 600mg as needed
Allergies : None
FHx : Father has hypertension ; mother has pseudogout
SHx : No smoking , occasional alcohol use

Physical examinations : Describe any positive and  negative findings relevant  to this patient¡¦s problem(s) . be careful to include only those parts of the examination performed in this encounter.

. Vital signs : Temperature , 38.3¡¦C(100.9F); blood pressure , 130/60 mmHg; pulse , 80/min; and respirations , 18/min
. HEENT : PERRLA , EOMI , no conjunctival hemorrhage
. Heart : RRR without murmurs , gallops , or rubs
. Musculoskeletal : Tender right knee , decreased ROM , no swelling or warmth , no other joint deformities
. Skin ; No rashes or lesions

Data interpretation : Based on what you have learned from the history and physical examination, list up to 3 diagnosis that might explain this patient¡¦s complaint(s) . List your diagnoses from most to least likely . For some cases , fewer than 3 diagnosis will be appropriate . the , enter the positive and negative findings form the history and the physical examination (if present) that support each diagnosis . Finally , list initial diagnostic (if any) you would order for each listed diagnosis (eg,restricted physical examination maneuvers, laboratory tests , imaging ECG , etc).

Diagnosis #1 : Osteoarthritis flare

History finding(s)
. History of chronic knee pain
. Acute worsening of right knee pain

Physical examination finding(s)
. Decreased ROM in right knee

Diagnosis #2 : Septic joints

History finding(s)
. Acute-onset right knee pain

Physical examination finding(s)
. Fever
. Decreased ROM in right knee

Diagnosis #3 : Acute crystal arthritis (Gout or pseudogout )

History finding(s)
. History of diuretic use
. Acute-onset right knee pain
. Family history of pseudogout Physical examination finding(s)
. Fever
. Decreased ROM in right knee

Diagnostic studies
. Right knee arthrocentesis
. Right knee x-ray
. ESR
. CBC with differential








µoªí¤å³¹®É¶¡2018/09/13 07:53am¡@IP: ¤w³]©w«O±K[¥»¤å¦@ 8671 ¦ì¤¸²Õ]¡@ 
JuanFe 

 

¸ê®Æ: ¦¹·|­û¥Ø«e¤£¦b½u¤W Male ¨°Às ¥Õ¦Ï®y
«Â±æ: 0
¾y¤O: ¾y¤O: 78957
¸gÅç: ¸gÅç: 35986
¨Ó¦Û: ¦t©z¤¤¡@blank
µo¤å: 1118 ½g
ºëµØ: 0 ½g
¦b½u: 47¤Ñ19®É22¤À57¬í
µù¥U: 2013/06/17
Message ¬d¬Ý¡@·j´M¡@³q°T¿ý¡@½Æ»s¡@¤Þ¥Î¡@¦^ÂФ峹¦^ÂС@[²Ä 30 ¼Ó]
 29 Case 29 scenario ( blurred vision )

Doorway information about patient

The patient is a 50-year-old man who comes to the office due to blurred vision

Vital signs

. Temperature : 36.7¡¦C (98.1F)
. Blood pressure : 160/90 mmHg
. Pulse : 70/min
. Respirations : 22/min

Basic differential diagnosis

. Diabetes mellitus
. Cataract
. Hypertensive retinopathy
. Glaucoma
. Macular degeneration
. Brain lesion
. Hyperviscosity syndorme (eg, polycythemia)
. Illicit drugs
. Temporal arthritis (usually starts unilaterally)
. Trauma to or infections of the eye (if unilateral)

¡X¡X¡X¡X¡X

Case 29 sim.pt. instruction

If the doctor asks you about anything other than these , just say ¡§ no ,¡¨ or provide an answer that a normal patient might give.

You are a 50-year-old man who comes to the office due to blurry vision

History of present illness

. Onset 2 month ago
. Objects are becoming increasingly blurry with no halos around them
. No headache , eye pain , or eye discharge
. Over-the -counter reading glasses have made only minimal improvement
. Last physician visit was 10 years ago

Do not volunteer this information unless asked :

Review of systems

. 10-lb weight loss
. Increased appetite , thirst , and urination
. No nausea or vomiting
. No muscle weakness
. No dizziness or loss pf consciousness
. No numbness or tingling in the extremities

Past medical / family / social history

. No prior ,medical issue , surgeries , or hospitalizations
. Medications : None
. No medication allergies
. Father has hypertension and motor has diabetes(you have no siblings)
. Married;live with wife
. Occupation ;Truck driver
. Tobacco ; 1 pack a day for the last 30 years
. Alcohol : Occasional beer
. Recreational drugs : None

Physical examination

HEENT :
. PERRLA , EOMI
. Funds show no hemorrhage or AV nicking

Neck : . Supple
. No lymphadenopathy or thyromegaly
. No bruits

Heart :
. Regular rate and rhythm without murmurs , gallops, or rubs

Extremities :
. Pulse 2+ in bilateral lower extremities

Neurologic :
. Motor strength 5/5 bilaterally
. Sensation grossly intact bilaterally

¡X¡X¡X¡X¡X

Case 29 sim. pt. checklist

Following the encounter , check which of the following items were performed by the examinee

History of present illness/ review of systems

. Asked about the onset of symptoms
. Asked whether symptoms were in 1 or both eyes
. Asked about the severity and course over time , Asked about the eye discharge
. Asked about halos around the light
. Asked about eye pain
. Asked about any headache
. Asked about nausea and vomiting
. Asked bout any weakness or sensory changes in the areas and legs . Asked about excessive thirst and urination
. Asked about changes in the appetite and weight

Past medical /family/social history

. Asked about similar episodes in the past
. Asked about past medical issues (especially diabetes and hypertension)
. Asked about current medications
. Asked about medication allergies
. Asked about family health
. Asked bout tobacco , alcohol , and recreational drug use
. Asked bout occupation

Examination

. Washed heads before examination
. Examined without gown , not through gown
. Examined eyes , including extra ocular movements , pupillary reflexes , and ophthalmoscope examination
. Did a neurological examination with emphasis on sensory examination
. Auscultated heart and carotid arteries
. Examined peripheral pulses

Counseling

. Explained physical findings and possible diagnosis (especially diabetes)
. Discussed dietary changes and weight reduction
. Explained further evaluation

Communication skills and professional conduct

. Knocked before entering the room
. Introduced self and greeted you warmly
. Used your name to address you
. Paid attention to what you said and maintained good eye contact
. Asked opened questions
. Asked non-leading questions . Asked one question at a time
. Listened to what you said without interrupting me
. Used plain English rather than technical jargon
. Used appropriate transition sentences
. Used appropriate draping techniques
. Summarized the history and explained physical findings
. Expressed empathy and gave appropriate reassurances
. Asked whether you have any concerns/questions

Differential diagnosis

. Diabetic retinopathy and /or diabetes osmotic changes in the lens
. Hypertensive retinopathy
. Cataracts
. Glaucoma
. Macular degeneration

Diagnostic study/studies

. Fasting blood glucose and /or hemoglobin A1c
. Urinalysis for microscopic proteinuria
. Lipid profile
. Carotid ultrasound

¡X¡X¡X¡X¡X

Case 29 clinical summary

Clinical Skills Evaluation
Case 29 Patient Note

The following represents a typical note for this patient encounter . the detail may vary depending on the information given by the simulated patient.

History : Describe the history you just obtained form this patient . Include only information (Pertinent positives and negatives)  relevant to this patient¡¦s problem(s).

. 50-yo man with 2 months of blurry vision , polyuria , polydipsia , polyphagia , and 4.5-kg (10-lb ) weight loss
. Has not seen a doctor in 10 years
. Objects blurry without complete loss of vision or halos around it

ROS : No nausea , vomiting , headache , arm/leg weakness , eye discharge , eye pain , dizziness , loss of consciousness, or numbness or tingling in the extremities
PMHx : None
PSHx : None
Meds : none
Allergies : None
FHx : Father has hypertension , mother has diabetes
SHx : 1 PPD smoker for past 30 years , occasional alcohol use

Physical examinations : Describe any positive and  negative findings relevant  to this patient¡¦s problem(s) . be careful to include only those parts of the examination performed in this encounter.

. Vital signs : Temperature , 36.7¡¨C(98.1F) ; blood pressure , 160/90 mmHg; pulse , 70/min; and respirations , 16/min
. HEENT : PERRLA , EOMI < funds without hemorrhages or AV nicking
. Neck : Supple without lymphadenopathy , thyromegaly, or bruits
. Extremities : Pulses 2+ in bilateral lower extremities
. Neurologic : Motor 5/5 bilaterally , sensory grossly intact bilaterally

Data interpretation : Based on what you have learned from the history and physical examination, list up to 3 diagnosis that might explain this patient¡¦s complaint(s) . List your diagnoses from most to least likely . For some cases , fewer than 3 diagnosis will be appropriate . the , enter the positive and negative findings form the history and the physical examination (if present) that support each diagnosis . Finally , list initial diagnostic (if any) you would order for each listed diagnosis (eg,restricted physical examination maneuvers, laboratory tests , imaging ECG , etc).

Diagnosis #1 : Diabetic retinopoathy

History finding(s)
. Polyuria , polydipsia , polyphagia
. Weight loss
. Blurry vision

Physical examination finding(s)
. None

Diagnosis #2 : Hypertension retinopathy

History finding(s)
. Blurry vision

Physical examination finding(s)
. BP 160/90 mmHg

Diagnosis #3 : Glaucoma

History finding(s)
. Decreased vision

Physical examination finding(s)
. None

Diagnostic studies

. Fasting blood glucose and hemoglobin A1c
. Eye examination to mesure pressure
. Lipid profile
. CBC with differential







µoªí¤å³¹®É¶¡2018/09/13 07:54am¡@IP: ¤w³]©w«O±K[¥»¤å¦@ 7629 ¦ì¤¸²Õ]¡@ 

 ¦@ 3 ­¶ 9 7 [ 1 2 3 ] 8 :

§Ö³t¦^ÂÐ¥DÃD: Sample cases
±z¥Ø«eªº¨­¥÷¬O¡G ³X«È ¡A­n¨Ï¥Î¨ä¥L·|­û¨­¥÷¡A½Ð¿é¤J·|­û¦WºÙ©M±K½X¡C¥¼µù¥U³X«È½Ð¿é¤Jºô¦W¡A±K½X¯dªÅ¥Õ¡C
¿é¤J·|­û¦WºÙ©M±K½X: ·|­û¦WºÙ: ¨S¦³µù¥U¡H¡@±K½X: §Ñ°O±K½X¡H
¤W¶Çªþ¥ó©Î¹Ï¤ù (³Ì¤j®e¶q 9000KB)
¥Ø«eªþ¥ó:(¦p¤£»Ý­n¬Y­Óªþ¥ó¡A¥u»Ý§R°£¤º®e¤¤ªº¬ÛÃö [UploadFile ...] ¼ÐÅÒ§Y¥i) [§R°£]
¿ï¶µ

¨Ï¥Î LeoBBS ¼ÐÅÒ¡H
Åã¥Ü±zªºÃ±¦WÀÉ¡H
¨Ï¥Îªí±¡²Å¸¹Âà´«¡H
¨Ï¥Î¦r«¬¼Ë¦¡Âà´«¡H

¡@¡@¡@¡@§Ö³t¤Þ¥Î²Ä ¼Ó¼hªº¦^ÂÐ
 ³»ºÝ¡@¥[¨ì"§Úªº³Ì·R" ¥DÃDºÞ²z¡G Á`©T³» ¨ú®øÁ`©T³» °Ï©T³» ¨ú®ø°Ï©T³» ©T³» ¨ú®ø©T³»
¥[­« ¨ú®ø¥[­« ºëµØ ¨ú®øºëµØ ´£¤É Âê©w ¸ÑÂê §R°£ §R°£¦^ÂÐ ²¾°Ê


© ¤¤¤åª©Åv©Ò¦³¡G ¶ð¤º´Óª«¶é¡@ÁcÅ骩Åv©Ò¦³¡G ¼¯¥§ºô
µ{¦¡ª©Åv©Ò¦³¡G ¹p¶Æ¬ì§Þ¡@ µ{¦¡Â½Ä¶¡Gauron¡@ª©¥»¡G LeoBBS X Build041101
 

¥»½×¾Â¨¥½×¯ÂÄݵo¨¥ªÌ­Ó¤H·N¨£¡A»P ¶ð¤º´Óª«¶é ¥ß³õµLÃö